Download as pdf or txt
Download as pdf or txt
You are on page 1of 106

Joint Admission and Matriculation Board

JAMB
MATHEMATICS
Past Questions

Years: 2005 - 2021

Send us a message at pdfmadeazy.com.ng


for more past
questions and answers download.
Or WhatsApp us on 08134857339
Mathematics 2005
1. If M represents the median and D the mode of the 10. If x + 2 and x – 1 are factors of the expressions lx +
measurements 5, 9, 3, 5, 8 then (M,D) is 2kx2 + 24, find the values of l and k
A. (6,5) B. (5,8) C. (5,7) A. l = -6, k = -9 B. l=-2,k =1 C. l=-2, k=-1
D. (5,5) E. (7,5) D. l = 0, k = 1 E. l = 6, k = 0

2. A construction company is owned by two partners X 11. Make T the subject of the equation
and Y and it is agreed that their profit will be divided in av = 3 2V + T
the ratio 4:5. at the end of the year. Y received #5,000 1- V a 2T
more than x. what is the total profit of the company for
the year? A. 3av/(1-v) B. 2v(1-v)2 - a2v2/2a2v2 - (1-V)2
A. #20,000.00 B. P’0#25,000.00 C. #30,000.00 C. 2v(1 - v)2 + a3v2/ 2a2v2 + (1 - v)2
D. #15,000.003 E.#45,000.00 D. 2v(1 - v)2 - a4v3/2a3v3 - (1 - v)3
E. 2v(1-v)3 - a4v3/2a3v3 + (1-v)3
3. Given a regular hexagon, calculate each interior angle
of the hexagon. 12.
A. 600 B. 300 C. 1200 Additional
Mathematics
0 0
D. 45 E. 135 (2x-24)
O
Biology
O
(3x-18)
x
O

Geography
4. Solve the following equations (2x+12)
O

4x – 3 = 3x + y = 2y + 5x – 12 O
(x+12) History
French
A. 4x = 5, y= 2 B. x= 2, y= 5 C. x =-2, y= -5
D. x = 5, y= -2 E. x = -5, y= -2
In a class of 60 pupils, the statistical distribution of the
5. If x = 1 is root of the equation number of pupils offering Biology, History, French,
x3 – 2x2 – 5x + 6, find the other roots Geography and Additional Mathematics is as shown in
A. -3and2 B. –2 and 2 C. 3 and–2 the pie chart above. How many pupils offer Additional
D. 1 and 3 E. –3and 1 Mathematics?
A. 15 B. 10 C. 18
6. If x is jointly proportional to the cube of y and the D. 12 E. 28
fourth power of z. In what ratio is x increased or
decreased when y is halved and z is doubled? 13 The value of (0.303)3 – (0.02)3 is
A. 4:1 increase B. 2:1 increase C. 1:4 decrease A. 0.019 B. 0.0019 C. 0.00019
D. 1: 1 no change E. 3: 4 decrease D. 0.000019 E. 0.000035

7. P Q 14. y varies partly as the square of x and y partly as the


O
O
60 inverse of the square root of x. write down the
45
expression for y if y = 2 when x = 1 and y = 6 when x =
8 cm 4
A. y = 10x2 + 52 B. y = x2 + 1
S R 31 31x x
C. y = x2 + 1 D. y= x2 + 1 E. y = 10 (x2 + 1 )
In the above figure PQR = 600, QPR = 900, PRS = 900, x 31 31 x 31( x)
RPS = 450, QR= 8cm. Determine PS 15. Simplify (x – 7) / (x2 – 9) ( x2 – 3x)/( x2 - 49)
A. 23cm B. 46cm C. 2cm A. x/(x-3)(x+7) B. (x+3)(x+7)/x C. x/(x-3) (x-
D. 86cm E. 8cm 7)
D. x/(x+3)(x+7) E. x/(x+4)(x+7)
8. Given that cos z = L, where z is an acute angle find an
expression for Co +Z - cosecz 16. The lengths of the sides of a right-angled triangle at (3x
sec Z + tan z + 1)cm, (3x - 1)cm and x cm.
A. l - L B. L2-L2 C. -L-L A. 2 B. 6 C. 18
1+L L2+L-1 (C1+L) +L D. 12 E. 0

D. L. E. L-(L2-1) 17. The scores of a set of a final year students in the first
(L1+L2) +L2 1+ 1 - L2+ 1 - L2 semester examination in a paper are
41,29,55,21,47,70,70,40,43,56,73,23,50,50. find themedian
9. If 0.0000152 x 0.00042 =Ax 108, where of the scores.
1 £ A < 10, find A and B. A. 47 B. 481/2 C. 50
A. A= 9, B = 6`.38 B. A= 6.38, B = -9 C. A= 6.38, B = 9 D. 48 E. 49
D. A= 6.38, B = -1 E. A= 6.38, B = 1
y
18. 12
A. –28,7 B. 6,-28 C. 6,-1
9 D. –1, 7 E. 3,2
6
3 25. Find the missing value in the following table.
-3 -2 -1 32 1
-3
-6
x -2 -1 0 1 2 3
y = x3 - x + 3
O
-9 3 3 3 9 27
-12
-15
A. -3 B. 3 C. –9
D. 13 E. 9
x
Which of the following equations represents the above 26.
graph?
A. y= 1+ 2x + 3x2 B. y= 1 – 2x+ 3x2 C. y= 1 + 2x3x2
D. y= 1 – 2x – 3x2 E. y= 3x2+2x-1 O
H xO
G
19. x
130O

If O is the centre of the circle in the figure above. Find


the value of x
O
A. 50 B. 260 C. 100
30 D. 65 E. 130
K F
27. Find the angle of the sectors representing each item in
The above figure FGHK is a rhombus. What is the value a pie chart of the following data. 6,10,14,16,26
of the anglex? A. 150,250,350,400,650, B.600,1000,1400,1600,2600
A. 900 B. 300 C. 1500 C. 60,100,140,160,260, D.300,500,700,800,1300
D. 120S0
E. 60 0 E. None ofthe above
0-8 m
20. P 28. The scores of 16 students in a Mathematics test are
0
65,65,55,60,60,65,60,70,75,70,65,70,60,65,65,70
R What is the sum of the median and modal scores?
2m A. 125 B. 130 C. 140
Q D. 150 E. 137.5
30O
29. The letters of the word MATRICULATION are cut and
PQRS is a desk of dimensions 2m x 0.8m which is inclined put into a box. One of the letter is drawn at random from
at 300 to the horizontal. Find the inclination of the the box. Find the probability of drawing a vowel.
diagonal PR to the horizontal.
A. 23035’ B. 300 C. 15036’ A. 2/13 B. 5/13 C. 6/13
0 0
D. 10 E. 10 42’ D. 8/13 E. 4/13

21. Find x if(x base 4)2 = 100 1000base 2 30. Correct each of the number 59.81789 and 0.0746829 to
A. 6 B. 12 C. 100 three significant figures and multiplythem, giving your
D. 210 E. 110 answer to three significantfigures.
A. 4.46 B. 4.48 C. 4.47
22. Simplify log a1/2 + 1/4log a – 1/12log a7 D. 4.49 E. 4.50
10 10 10
A. 1 B. 7/6log10a C. 0
D. 10 E. a 31. If a rod of length 250cm is measured as 255cm longer in
error, what is the percentage error in measurement?
23. If w varies inversely as V and u varies directly as w3, A. 55 B. 10 C. 5
find the relationship between u and V given that u = 1, D. 4 E. 2
when V = 2
A. u= 8V3 B. u= 2 V C. V = 8/u2 32. If (2/3)m (3/4)n = 256/729, find thevalues of m and n
2 3
D. V= 8u E. U= 8/v A. m=4, n = 2 B. m=-4, n =-2 C. m=-4,n =2
D. m=4,n =-2 E. m=-2,n =4
24. Solve the simultaneous equations for x
x2 + y – 8 = 0 33. Without using tables find the numerical value of log749
y + 5x – 2 = 0 + log7(1/7)
A. 1 B. 2 C. 3
D. 7 E. 0
34. Factorize completely 81a4 – 16b4
A. (3a + 2b) (2a – 3b) (9a2 + 4b2) 41. In the figure below find PRQ
B. (3a - 2b) (2a – 3b) (4a2 - 9b2)
C. (3a - 2b) (3a – 2b) (9a2 + 4b2)
D. (3a - 2b) (2a – 3b) (9a2 + 4b2)
235
E. (3a - 2b) (2a – 3b) (9a2 - 4b2)
o
R
35. One interior angle of a convex hexagon is 1700 and
each of the remaining interior angles is equal to x0. find
x Q
A. 1200 B. 1100 C. 1050 P
1 0
D. 1020 E. 1000 A. 66 / B. 621/ 0 C. 1250
2 2
D. 1050 E. 650
36. PQRS is a cyclic quadrilateral in which PQ = PS. PT is a
tangent to the circle and PQ makes and angle 500 with
the tangent as shown in the figure below. What is the 42. Simplify 27a9/8
size of QRS? A. 9a2/2 B. 9a3/2 C. 2/3a2
R D. 2/3a2 E. 3a3/2

43. Okro
14.5
Beans
kg
S Q 14.5 kg Rice
45.4
O Yams kg
50
T 184.5 kg
P
0
A. 50 B. 400 C. 1100
D. 800 E. 1000
The farm yields of four crops on a piece of land in
37. A ship H leaves a port P and sails 30km due South. Ondo are represented on the pie chart above. What is
Then it sails 60km due west. What is the bearing of H the angle of the sector occupied by Okro in the chart?
from P? A. 911/2 0 B. 191/ 0 C. 331/3 0 3
A. 26034’ B. 243026’ C. 116034’ D. 110 E. 910
0 0
D. 63 26’ E. 240
O
44. (x+3y)
38. In a sample survey of a university community the O
following table shows the percentage distribution of P 45
Q R
the number of members perhousehold. yO
O
No of members (3x+y)
per household 12 3 4 56 7 8 Total
Number of
households 3 12 15 28 21 10 7 4 100
In the figure above, PQR is a straight line. Find the
A. 4 B. 3 C. 5 values of x and y
D. 4.5 E. None A. x = 22.50 and y = 33.750
B. x = 150 and y = 52.50
39. On a square paper of length 2.524375cm is inscribed a C. x = 22.50 and y = 45.00
square diagram of length 0.524375. find the area of the D. x = 56.250 and y = 11.50
paper no covered by the diagram correct to 3 significant E. x = 18.0 and y = 56.50
figures.
45. PQR is the diameter of a semicircle RSP with centre at
A. 6.00cm2 B. 6.10cm2 C. 6.cm2
2 2 Q and radius of length 3.5cmc. if QPT = QRT = 600. Find
D. 6.09cm E. 4.00cm
the perimeter of the figure (PTRS p = 22/7)
S
40. If f(X) = 1 + x - 1 find f(1-x)
x-1 x2-1

P
A. 1/x + 1/(x+2) B. x +1/(2x -1) R
60O O 60O

C. -1/x - 1/(x-2) D. -1/x + 1/(x2-1)

A. 25cm B. 18ccm C. 36cm


D. 29cm E. 25 5 cm
46. In a triangle PQR, QR = 3cm, PR = 3cm, PQ = 3cm and 49. Q 30
O

PQR = 300. find angles P and R


A. P = 600 and R = 900
B. P = 300 and R = 1200
O
C. P = 900 and R = 600
D. P = 600 and R = 600 xO
E. P = 450 and R = 1050 T
O

x O 2x
47. P
S R

130
O
O
In the figure above PT is a tangent to the circle with
100 centre O. if PQT = 300. find the value of PTO
A. 300 B. 150 C. 240
D. 120 E. 600
P Q
50 A man drove for 4hours at a certain speed, he then
In the above diagram if PS = SR and PQ//SR. what is the doubled his speed and drove for another 3 hours.
size of PQR? Altogether he covered 600km. At what speed did he
A. 250 B. 500 C. 550 drive for the last 3 hours?
D. 65 0
E. 75 0 A. 120km/hr B. 60km/hr C. 600/7km/hr
D. 50km/hr E. 100km/hr.
48. Find the mean of the following
24.57,25.63,25.32,26.01,25.77
A. 25.12 B. 25.30 C. 25.26
D. 25.50q E. 25.73

Mathematics 2006

1. Simplify (2/3 – 1/5) – 1/3 of 2/5 6. A man invested a total of #50,000 in two companies. If
3 – 1/ 1/2 these companies pay dividend of 6% and 8%
A. 1/7B. 7 C. 1/3 respectively, how much did he invest at 8% if the total
D. 3 E. 1/5 yield is #3.700?
A. #15,000 B. #29,600 C. #21,400
2. If 263 + 441 = 714, what number base has been used? D. #27,800 E. #35,000
A. 12 B. 11 C. 10
D. 9 E. 8 7. Thirty boys and x girls sat for a test. The mean of the
boys’ scores and that of the girls were respectively 6
3. 0.00014323/1.940000 = k x 10n where 1 £ k < 10 and n is and 8. find x if the total score was 468.
a whole number. The values of K and are A. 38 B. 24 C. 36
A. 7.381 and –11 B. ......... 2.34 and10 D. 22 E. 41
C. 3.87 and 2 D. ......... 7.831 and–11
E........... 5.41 and–2 8. The cost of production of an article is made up as
follows Labour #70
4. P sold his bicycle to Q at a profit of 10%. Q sold it to R Power #15
for #209 at a loss of 5%. How much did the bicycle cost Materials #30
P? Miscellaneous #5
A. #200 B. #196 C. #180 Find the angle of the sector representing labour in a pie
D. #205 E. #150 chart.
A. 2100 B. 1050 C. 1750
0 0
5. If the price of oranges was raised by 1/2k per orange, D. 150 E. 90
the number of oranges customer can buy for #2.40 will
9. Bola chooses at random a number between 1 and 300.
be less by 16. What is the present price of an orange?
What is the probability that the number is divisible by
A. 21/2k B. 31/2k C. 51/2 k
D. 20k E. 1
21 / k 4?
2 A. 1/3 B. ¼ C. 1/5
D. 4/300 E. 1/300
10. Find without using logarithm tables, the value of 19. In a racing competition. Musa covered a distance of 5xkm
Log327 – Log1/464 in the first hour and (x + 10)km in the next hour. He was
Log31/81 second to Ngozi who covered a total distance of 118km
A. 7/4 B. –7/4 C. –3/2 in the two hours. Which of the following inequalities is
D. 7/3 E. –1/4 correct?
A. 0 < -x < 15 B. –3 < x < 3
11. A variable point P(x, y) traces a graph in a two C. 15 <x < 18 D. 0 < x < 15
dimensional plane. (0, -3) is one position of P. If x E. 0 < x < 18
increases by 1 unit, y increases by 4 units. The equation
of the graph is 20. 2x + 3y = 1 and y = x – 2y = 11, find (x + y)
A. -3 = y + 4/ x + 1 B. 4y = -3 + x A. 5 B. –3 C. 8
C. y/x = -3/4 D. y + 3 = 4x D. 2 E. 2
E. 4y = x + 3
21. Tunde and Shola can do a piece of work in 18days.
12. A trader in a country where their currency ‘MONT’ (M) Tunde can do it alone in x days, whilst Shola takes 15
is in base five bought 103(5) oranges at M14(5) each. If days longer to do it alone. Which of the following
he sold the oranges at M24(5) each, what will be his equations is satisfied by x?
gain? A. x2– 5x– 18 = 0 B. x2 – 20x+360 =0
2 2
A. M103 (5) B. M1030(5) C. M102 (5) C. x -21x–270=0 D. 2x +42x–190=0
D. M2002(5) E. M3032(5) E. 3x2–31x+150=0

13. Rationalize 22. If fx) = 2(x - 3)2 + 3(x - 3) – 4 and g(y) = 5 + y, find g(f(3))
(55 - 75)(/7 - 5 and g{f(4)}
A. -235 B. 47 - 65 C. -35 A. 3 and 4 B. –3 and 4
D. 47 - 85 E. 35 C. –3 and –4 D. 3 and –4
14. Simplify E. 0 and 5
3n – 3 n – 1
33 x 3n – 27 x 3n– 1 23. The quadratic equation whose roots are 1 - 13 and 1 +
A. 1 B. 0 C. 1/27 13 is
D. 3n – 3n – 1 E. 2/27 A. x2+(1-13)x+1+13=0
B. x2+(1-13)x+1-13=0
15. p varies directly as the square of q an inversely as r. if C. x2+2x+12=0 D. x2 –2x+ 12=0
2
p = 36, when q = 3 and r = p, find p when q = 5 and r = 2 E. x – 2x–12=0
A. 72 B. 100 C. 90
D. 200 E. 125 24. Find a factor which is common to all three binomial
expressions
16. Factorise 6x2 – 14x - 12 4a2 – 9b2, a3 + 27b3, (4a + 6b)2
A. 2(x + 3) (3x - 2) B. 6(x - 2) (x + 1) A. 4a + 6b B. 4a – 6b
C. 2(x - 3) (3x + 2) D. 6(x + 2) (x - 1) C. 2a + 3b D. 2a – 3b
E. (3x + 4) (2x + 3) E. none

17. A straight line y = mx meets the curve y = x2 – 12x + 40 25.


in two distinct points. If one of them is (5,5), find the P
5 cm
other
A. (5,6) B. (8,8) C. (8,5) Q
D. (7,7) E. (7,5)
S
18. The table below is drawn for a graph y = x2 – 3x + 1 11 cm

T
R 4 cm
x -3 -2 -1 0 1 2 3
y=x2 - 3x + 1 1 -1 3 1 -1 3 1
What is the volume of the regular three dimensional
From x = -2 to x = 1, the graph crosses the x-axis in the figure drawn above?
range(s) A. 160cm3 B. 48cm3 C. 96cm3
3 3
A. -1 < x < 0 and 0 < x < 1 D. 120cm E. 40cm
B. -2 < x < -1 and 0 < x < 1
C. -2 < x < -1 and 0 < x < 1 26. If (x - 2) and (x + 1) are factors of the expression x3 + px2
D. 0 < x < 1 E. 1<x<2 + qx + 1, what is the sum of p and q?
A. 0 B. –3 C. 3
D. –17/3 E. –2/3
27. A cone is formed by bending a sector of a circle having 34.
an angle of 2100. Find the radius of the base of the cone
if the diameter of the circle is base of the cone if the
diameter of the circle is 12cm r r
A. 7.00cm B. 1.75cm C. Ö21cm
D. 3.50cm E. 2Ö21cm
O
60
O O
28. X r 60 60 r
120 Find the area of the shaded portion of the semi – circular
figure above.
A. r2/4(4p - 3 3) B. r2/4(2p + 3 3)
3 cm 2
C. 1/2r p D. 1/8r 3
E. r2/8(4p + 3 3)

35. P
Y Z
5 cm 44
O

Using XYZ in the figure above find XYZ r


A. 290 B. 31020’ C. 310 O
0 20
D. 31 18’ E. 590
Q
29. The sides of a triangle are (x + 4)cm, x cm and (x- 4) cm S
respectively. If the cosine of the largest angle is 1/5,
find the value of x
A. 24cm B. 20cm C. 28cm y
D. 88/7ccm E. 0cm R
In the figure above QRS is a line, PSQ = 350 SPR = 300
30. If a = 2x/1 – x and b = 1 + x / 1 – x and O is the centre of the circle find OQP
then a2 – b2 in thesimplest form is A. 350 B. 300 C. 1300
0
A.3x+1/(x-1) B. 3x2-1/(x-1)2 D. 25 0
E. 65
2 2
C. 3x +1/(1-x) D. 5x2-1/(1-x)2
2 2
E. 5x -2x-1/(1-x) 36. If pq + 1 = q2 and t = 1/p – 1/pq
( x-1) express t in terms of q
31. Simplifty (1 + 1 ) (x+2) A. 1/p – q B. 1/ q – 1
( x+1) C. 1/q + 1 D. 1+q
E. 1/ 1- q
A. (x2 - 1)(x + 2) B. x2 (x + 2)/x + 1
C. x2 - (x + 2) D. 2x(x + 2) 37. The cumulative frequency function of the data below
E. 2x(x + 2)/x + 1 is given by the frequency y = cf(x). what is cf(5)?
Scores(n) Frequency(f)
32. Q 3 30
4 32
5 30
P R 6 35
7 20

V A. 30 B. 35 C. 55
D. 62 E. 92
W S
38. In the figure determine the angle marked y
A. 660 B. 1100 C. 260
D. 700 E. 440
T
P
In the figure above PQRSTW is a regular hexagon. QS
44O
intersects RT at V. calculate TVS.
A. 600 B. 900 C. 1200 r
0 0
D. 30 E. 80 20 O

33. Find the integral values of x which satisfy the Q


S
inequalities –3 < 2 –5x < 12
A. -2, -1 B. –2, 2 C. –1, 0
D. 0,1 E. 1,2 y
R
39. A right circular cone has a base radius r cm and a vertical 45. Bar Chart
2y0. the height of the cone is 10
A. r tan y0cm B. r sin y0cm
8

Frequency
0
C. r cot y cm D. r cos y0cm
E. 0
r cosec y cm 6
4
40. Two fair dice are rolled. What is the probability that
2
both show up the same number of point?
A. 1/36 B. 7/36 C. ½ 0
D. 1/3 E. 1/6 0-9 10-19 20-29 30-39 40-49 50-59
Marks
41. The larger value of y for which (y - 1)2 = 4y – 7 is
A. 2 B. 4 C. 6 The bar chart above shows the mark distribution in a
D. 7 E. 8 class test. Find the number of students in the class.
A. 9 B. 2 C. 60
42. y D. 30 E. 34

46. S T

O
135
P
O R

Q
x
In the figure above, O is the centre of circle PQRS and
PS//RT. If PRT = 1350, then PSQ is
A. 671/ 0 B. 450 C. 900
Find the x coordinates of the points of intersection of D. 333/2 0 E. 221/ 0
4 2
the two equations in the graph above.
A. 1,1 B. 0,-4 C. 4,9 47. XYZ is a triangle and XW is perpendicular to YZ at W.
D. 0,0 E. 0,4 if XZ = 5cm and WZ = 4cm, calculate XY.
A. 53cm B. 35cm C. 3Ö3cm
43. If sin q = x/y and 00 < q < 900 D. 5cm E. 6cm
then find 1/ tan q
A. x/y2 – x2) B. x/y X
C. y2 –n2
yx D. y2 – x2)/y2 – x2)
E. y2 – x2/y
5 cm
44.
P
Y 4 cm Z
6 cm
10 cm
R
Q 48. Measurements of the diameters in centimeters of 20
8 cm copper spheres are distributed as shown below

S T Class boundary in cm frequency


12 cm 3.35-3.45 3
3.45-3.55 6
In the figure above TSP =PRQ, QR = 8cm. PR = 6cm and 3.55-3.65 7
ST = 12cm. Find the length SP 3.65-3.75 4
A. 4cm B. 16cm C. 9cm
D. 14cm E. Impossible insufficient data What is the mean diameter of the copper sphere?
A. 3.40cm B. 3.58cm C. 3.56cm
D. 3.62cm E. 3.63cm
Use the instruction below to answer question49 and 50 49. What is the obtuse angle formed when the point U is
joined to Q?
A. 750 B. 1540 C. 1200
T U 0 0
D. 105 E. 125

50. What is the acute angle formed when the point V joined
to Q?
V R S A. 600 B. 300 C. 450
0 0
D. 90 E. 15

P O

Mathematics 2007
1. Arrange the following numbers in ascending order of A. 3/2 B. 2/3 C. 2
magnitude 6/7,13/15,0.865 D. 3 E. 18
A. 6/7 < 0.865 < 13/15
B. 6/7 < 13/15 < 0.865 8. Without using tables, evaluate Log24 + Log42 – Log255
C. 13/15 < 6/7 < 0.865 A. ½ B. 1/5 C.0
D. 13/15 < 0.865 < 6/7 D. 5 E. 2
E. 0.865 < 6/7 < 13/15
9. John gives one third of his money to Janet who has
2. A sum of money was invested at 8% per annum simple #105.00. He then finds that his money is reduced to
interest. If after 4years the money amounts to #330.00, one-fourth of what Janet now has. Find how much
find the amount originally invested. money John hadat first.
A. #180.00 B. #165.00 C. #150.00 A. #45.00 B. #48.00 C. #52.00
D. #200.00 E. #250.00 D. #58.00 E. #60.00

3. I the equation below, solve for x if all the numbers are in 10. Find x if Log9x = 1.5
base 2? 11/x= 1000/(x+ 101) A. 72.0 B. 27.0 C. 36.0
A. 101 B. 11 C. 110 D. 3.5 E. 24.5
D. 111 E. 10
11. Write h in terms of a =b(1 - ch)
4. List all integers satisfying the inequality (1-dh)
-2 < 2x – 6 < 4
A. 2,3,4,5 B. 2,3,4 C. 2,5 A h = (a - b) B. h = (a + b )
D. 3,4,5 E. 4,5 (ad- bc) (ad - bc)

5. Find correct to tow decimal places C. h = (ad - bc) D. h = (1 - b)


100 + 1/100 + 3/1000 + 27/10000 ( a - b) (d - bc)
A. 100.02 B. 1000.02
C. 100.22 D. 100.01 E. h = (b - a)
E. 100.51 (ad - bc)

6. Simplify 1/2 + 1 12. 221/2 % of the Nigerian Naira is equal to 171/10% of a foreign
1 currency M. what is the conversion rate of the M to the
2 + ------------- Naira?
1 A. 1M = 15/57N B. 1M = 21157/ N
2 - -------------- C. 18
1M = 1 /57N D. 1M = 381/4N
3
4 +1/5 E. 1M = 384 /4N
A. ¾ B. –1/3 C. 169/190
D. 13/15 E. 121/ 13. Find the values of p for which the equation x2 – (p - 2)x
169
+ 2p + 1 = 0 has equal roots
7. If three number p,q,r are in the ratio 6:4:5 find the value A. (0,12) B. (1,2) C. (21,0)
of (3q – q)/(4q + r) D. (4,5) E. (3,4)
14. If ex = 1 + x + x2/12 + x3/1.2.3 + ….. find 1/e1/2 25. If f(x - 2) = 4x2 + x + 7 find f(1)
A. 1 - x + x2 - x2 +... B. 1 + x + x2 + x2 A. 12 B. 27 C. 7
2 123 24 3 2 1.22 23.3 D. 46 E. 17
2 2
C. 1 + x + x - x +... D. 1 - x + x2 - x2 +
3 4.
2 1.2 2 3 2 1.22 23.3 26. In DXYZ, XY = 13cm, YZ = 9cm, XZ= 11cm and XYZ=
3 3 4
E. 1+ x + x - x + q0. find cos q0
1.2 12.4 12.63 A. 4/39
B. 43/39
5. (43 + 42) (43 - 42) (3 + 2) is equal to C. 209/286
A. 0 B. 43 + 42 D. 1/6
C. (42 - 43) (3 + 2) E. 43/78
D. 3 + 2 E. 1
27. Find the missing value in the table below
16. In a restaurant, the cost of providing a particular type
of food is partly constant and partly inversely x -2 -1 0 1 2 3
proportional to the number of people. If the cost per 2 O
y=x -x+3 3 3 3 9 27
head for 100people is 30k and the cost for 40 people is
60k, find the cost for 50people A. -32 B. –14 C. 40
A. 15k B. 45k C. 20k D. 22 E. 37
D. 50k E. 40k
28. Find the number of goals scored by a football team in
17. The factors of 9 – (x2 – 3x - 1)2 are 20matches is shown below
A. -(x - 4)(x + 1)(x- 1)(x - 2)
No . of goals 0 1 2 3 4 5
B. (x - 4)(x- 1)(x - 1)(x + 2)
No . of matches 3 5 7 4 1 0
C. -(x - 2)(x + 1)(x+ 2)(x + 4)
D. (x - 4)(x -3)(x- 2)(x + 1) What are the values of the mean and the mode
E. (x - 2)(x + 2)(x- 1)(x + 1) respectively?
A. (1.75, 5) B. (1.75, 2)
18. If 32y – 6(3y) = 27 find y C. (1.75, 1) D. (2,2)
A. 3 B. –1 C. 2 E. (2,1)
D. –3 E. 1
29. If the hypotenuse of a right angle isosceles triangle is
19. 2
Factorize abx + 8y – 4bx –2axy 2, what is the length of each of the other sides?
A. (ax - 4) (bx – 2y) B. (ax + b) (x – 8y) A. 2 B. 1/2 C. 22
C. (ax – 2y) (by– 4) D. (abx - 4) (x – 2y) D. 1 E. 2 -1
E. (bx - 4) (ax – 2y)
30. If two fair coins are tossed, what is the probability of
20. At what real value of x do the curves whose equations getting at least one head?
are y = x3 + x and y = x2 + 1 intersect? A. ¼ B. ½ C. 1
A. -2 B. 2 C. –1 D. 2/3 E. ¾
D. 0 E. 1
31. The ratio of the length of two similar rectangular blocks
21. 2
If the quadrilateral function 3x – 7x + R is a perfect is 2:3, if the volume of the larger block is 351cm3, then
square find R the volume of the other block is
A. 49/24 B. 49/3 C. 49/6 A. 234.00cm3 B. 526.50cm3
3
D. 49/12 E. 49/36 C. 166.00cm D. 729.75cm3
3
E. 104.00cm
22. Solve the following equation
2/(2r – 1) – 5/3 = 1/ (r + 2) 32. The bearing of bird on a tree from a hunter on the
A. (-1, 5/2) B. (-1, -5/2) ground is N720E. what is the bearing of the hunter from
C. (5/2, 1) D. (2, 1) the bird?
E. (1, 2) A. S180W B. S720W
0
C. S72 Eq D. S270E
23. Solve for (x,y) in the equations E. 0
S27 W
2x + y = 4: x2 + xy = -12
A. (6, -8); (-2,8) B. (3, -4); (-1, 4)
C. (8, -4); (-1, 4) D. (-8, 6);(8, -2)
E. (-4, 3);(4,-1)

24. Solve the simultaneous equations


2x – 3y + 10 = 10x – 6y = 5
A. x = 2 1/ , y = 3 1/ B. x = 31/ , y = 21/
2 3 2 3
1
C. x= 2 / ,y=3 D. x = 31/ , y =21/
4 2 5
E. x = 2 1/ , y = 2 1/
2 3
33. X 39. A solid sphere of radius 4cm has mass of 64kg. What will
be the mass of a shell of the same metal whose internal
25 and external radii are 2cm and 3cm respectively?
15 A. 5kg B. 16kg C. 19kg
D. 25kg E. 48kg
Y 8 Z
K 40. S

In D XYZ above, XKZ = 900, XK = 15cm, XZ cm and YK 145


R
= 8cm. Find the area of the D XYZ.
A. 180sq.cm B. 210sq.cm
C. 160sq.cm D. 320sq.cm P Q
E. 390sq.cm O

34. Without using tables. Calculate the value of 1 + sec230?


A. 21/3 B. 2 C. 11/3
D. ¾ E. 3/7
In the figure above POQ is the diameter of the circle
35. What is the probability that a number chosen at random PQRS. If PSR = 1450, find x0
from the integers between 1 and 10 inclusive is either a A. 250 B. 350 C. 450
0 0
prime or a multiple of 3? D. 55 E. 25
A. 7/10 B. 3/5 C. 4/5
D. ½ E. 3/10 41. N
M
36. Find the area of a regular hexagon inscribed in a circle
of radius 8cm.
K
A. 163cm2 B. 963cm2 L
2
C. 192.3cm D. 16cm2
2
E. 32cm
J I
37.X

H
N
In the figure above GHIJKLMN is a cube of side a. find
O
the length of HN
86 A. 3a B. 3a C. 3a2
O Q D. a2 E. a3
122
M
42. PQRS is a trapezium of area 14cm2 in which PQ//RS, if
PQ = 4cm and SR = 3cm, find the area of DSQR in cm2
A. 7.0 B. 6.0 C. 5.2
P
D. 5.0 E. 4.1

43.
Y
Q
In the figure above, MNOP is a cyclic quadrilateral,
MN and PQ are produced to meet at X and NQ and MP 0O
are produced to meet at Y. if MNQ = 860 and NQP= 1220,
find (x0,y0) 0O
A. (280,,360) B. (360 ,280) S P
R
C. (430,,610) D. (610 ,430)
0 0
E. (36 ,43 )

38. If cosq = 3/2 and 0 is less than 900, calculate


cot (90 - q) / sin2q In the figure PQ is the tangent from P to the circle QRS
A. 43/3 B. 43 with SR as its diameter. If PQR= q0, which of thefollowing
C. 3/2 D. 1/3 relationship 00 is correct.?
E. 2/3 A. q0 + f = 900 B. f0 = 900 - 200
0 0
C. q =f D. f0 = 200
0 0 0
E. q + 2f = 120
44. A bag contains 4 white balls and 6 red balls. Two Redballs 48. In a class of 120students, 18 of them scored an A grade in
are taken from the bag without replacement. What is Mathematics. If the section representing the A grade
the probability that they are both red? students on a pie chart has angle Z0 at the centre of the
A. 1/3 B. 2/9 C. 2/15 circle, what is Z?
D. 1/5 E. 3/5 A. 15 B. 28 C. 50
D. 52 E. 54
45. How many 2 2cm diameter discs can be cut out of a
sheet of cardboard 218 2p3/4cm long and p1/2cm wide? 49.
A. 49 B. 219 C. 217p3/4( 2p + 2)
O
D. 10 3/4
2 p (1 + 2) E. 29( 2 + 1) 80

46. Two points X and Y both on latitude 600S have O


40
longitudes 1470E and 1530W respectively. Find to the
nearest kilometre the distance between X and Y O

measured along the parallel of latitudes (Take 2 R = 4 x 20


104km, where R is the radius of the earth). xO
A. 28.850km B. 16.667km
C. 8.333km D. 6.667km In the figure above find the angle x
E. 3.333km A. 1000 B. 1200 C. 600
0
D. 110 E. 1400
47.
50. If a (x+1) - (x +1) = bx
( x-2 ) ( n+2)

120
O Find a simplest form
3 A. x2 – 1 B. x2 + 1 C. x2 + 4
3 D. 1 E. x2 - 4

In the figure above the area of the shaded segment is


A. 3p B. 9 3/4
C. 3(p - 3 3/4) D. 3( 3 - p)/4
E. p + 9 3/4

Mathematics 2008
1. Evaluate 5. Find the reciprocal of 2/3
(212)3 – (121)3 + (222)3 1/2 + 1/3
A. (313)3 B. (1000)3
C. (1020)3 D. (1222)3 A. 4/5 B. 5/4
C. 2/5 D. 6/7
2. If Musa scored 75 in Biology instead of 57, his
average mark in four subjects would have been 60. 6. Three boys shared some oranges. The first receive 1/3
what was his total mark? of the oranges, the second received 2/3 of the remainder,
A. 282 B. 240 if the third boy received the remaining 12 oranges. How
C. 222 D. 210 many oranges did they share?
A. 60 B. 54
3. Divide the L.C.M. of 48, 64 and 80 by their H.C.F C. 48 D. 42
A. 20 B. 30
C. 48 D. 60 7. If P = 18, Q = 21, R = -6 and S = -4 calculate (P - Q) + S2
A. -11/216 B. 11/216
4. Find the smallest number by which 252 can be C. –43/115 D. 41/116
multiplied to obtain a perfect square
A. 2 B. 3
C. 5 D. 7
8. Simplify 0.03 x 4 x 0.00064 20. Factorize x2 + 2a + ax + 2x
0.48 x 0.012 A. (x + 2a)(x + 1) B. (x + 2a)(x - 1)
A. 3.6 x 102 B. 36 x 102 C. (x2 - 1)(x + a) D. (x + 2)(x + a)
C. 3.6 x 103 D. 3.6 x104
21. Solve the equation 3x2 + 6x – 2 = 0
9. Udoh deposited #150 00 in the bank. At the end of 5 A. x = -1,3/3 B. x = -1,15/3
years the simple interest on the principal was #55 00. C. x =-2, 23/3 D. x = -2, 215/3
At what rate per annum was the interest paid?
A. 11% B. 71/3 % 22. Simplify. 1/ 5x +5 + 1/7x + 7
C. 5% D. 31/ % A. 12/35+7 B. 1/35(x+1)
2

10. A number of pencils were shared out among Bisi, Sola C. 12x/35(x+1) D. 12/35x +35
and Tunde in the ratio 2:3:5 respectively. If Bisi got 5,
how many were shared out? 23. The curve y = -x2 + 3x + 4 intersects the coordinate axes
A. 15 B. 25 at
C. 30 D. 50 A. (4,0)(0,0)(-1,0) B. (-4,0)(0,4)(1,1)
C. (0,0)(0,1)(1,0) D. (0,4)(4,0)(-1,0)
11. The ages of Tosan and Isa differ by 6 and the product
of their ages is 187. write their ages in the form (x, y), 24. Factorize (4a + 3)2 – (3a - 2)2
where x >y A. (a + 1)(a+ 5) B. (a - 5)(7a - 1)
A. (12, 9) B. (23, 17) C. (a + 5)(7a + 1) D. a(7a + 1)
C. (17, 11) D. (18, 12)
25. If 5(x + 2y) = 5 and 4(x + 3y) = 16, find 3(x +y)
12. In 1984, Ike was 24 years old and is father was 45 years A. 0 B. 1
old in what year was Ike exactly half his father’s age? C. 3 D. 27
A. 1982 B. 1981
C. 1979 D. 1978 26. Simplify 1/ x - 2 + 1/ x + 2 + 2x / x2 - 4

13. Simplify ( 1 1 ) x -1/ A. 2x/ (x-2) (x+2) (x2 - 4) B.2x/x2 - 4


      
C. x/x2- 4 D. 4x/ x2 - 4

A. 3/5 B. –2/3
27. Make r the subject of the formula
C. –2 D. –1
S = 6/v - w/2
14. Find n if Log 4 + Log Z – Log n = -1 A. V = 6 = 12 B. v = 12
A. 10 2
2
B. 2 14 S2 2w 252 - w
C. v = 12 - 2s D. v = 12
C. 27 D. 28
w 2s2 + w
15. (91/3 x 27-1/2) / (3-1/6 x 3-2/3)
28. Find the values of x which satisfy theequation
A. 1/3 B. 1 16x – 5x 4x + 4 = 0
A. 1 and 4 B. –2 and 2
C. 3 D. 9
C. 0 and 1 D. 1 and 0

16. If x varies directly as y3 and x = 2 when y = 1, find x


29. a/b –c/d = k, find the value of
when y = 5
A. 2 B. 10 (3a2 – ac + c2)/(3b2 – bd + d2) in term of k
C. 125 D. 250 A. 3k2 B. 3k –k2
C. 17k2/4 D. k2
17. Factorize completely.
30. At what point does the straight line y = 2x + 1 intersect
3a + 125ax3
2 the curve y = 2x2 + 5x – 1?
A. (2a+ 5x )(4 + 25ax)
A. (-2,-3) and(1/2, 2) B. (-1/2 0) and (2, 5)
B. a(2 + 5x)(4 – 10x + 25ax2)
C. (1/2, 2) and(1, 3) D. (1, 3) and (2, 5)
C. (2a + 5x)(4 - 10ax + 25ax2)
D. a(2 + 5x)(4 + 10ax + 25ax2)
31. A regular polygon on n sides has 1600 as the size each
interior. Find n.
18. If y = x/(x – 3) + x/(x + 4) find y when x = -2 A. 18 B. 16
A. -3/5 B. 3/5
C. 14 D. 12
C. –7/5 D. 7/5

32. If cos q = a/b, find 1 +tan2q


19. Find all the numbers x which satisfy the inequality 1/
A. b2/a2 B. a2/b2
3(x + 1) – 1 > 1/5 (x + 4)
C. (a2 + b2) / (b2 – a2) D. (2a2 + b2)/ (a2 + b2)
A. x< 11 B. x< -1
C. x> 6 D. x> 11
33. In the diagram below, PQ and RS are chords of a circle 38. An arc of circle of radius 6cm is 8cm long. Find the area of
centre O which meet at T outside the circle. If TP = the sector.
24cm, TQ = 8cm and TS = 12cm, find TR. A. 51/3 cm2 B. 24cm2
2
C. 36cm D. 48cm2
Q
P T 39.X
O
R
S 4 3

A. 16cm B. 14cm
C. 12cm D. 8cm Y Z
6
34. The angle of elevation of the top of a vertical tower 50 In XYZ above, determine the cosine of angle Z
metres high from a point X on the ground is 300. From a A. ¾ B. 29/36
point Y on the opposite side of the tower, the angle of C. 2/3 D. ½
elevation of the top of the tower is 600. find the distance T
between the points X and Y. 40. S
A. 14.43m B. 57.73m
C. 101.03m D. 115.47m
O

35. A girl walk 45 metres in the direction 0500 from a point Q 20


O 35
Q R
to a point X. She then walks 24metres in the direction
1400 from X to a point Y. How far is she then from Q?
A. 69m B. 57m In the figure above PQT is isosceles. PQ = QT. SRQ
C. 51m D. 21m = 350, TQ = 200 and PQR is a straight line. Calculate
TSR.
36. A. 200 B. 550
C. 75 D. 1400

11 m 41. Find the total surface are of a solid cone of radius 2 3cm
and slanting side 4 3cm
A. 83cm2 B. 24cm2
2
P R C. 153cm D. 36cm2

6m 12 m 42. If U and V are two distinct fixed points and W is a


variable point such that UWV is a straight angle. What
8m Q is the locus of W?
A. The perpendicular bisector of UV
The figure is a solid with the trapezium PQRS as its B. A circle with UV as radius
uniform cross-section. Find its volume C. A line parallel to the line UV
A. 102m3 B. 576m3 D. A circle with the line UV as the diameter
3
C. 816m D. 1056m3
43.
37.
Q

x
P
T
O
O
65
O
35
R

PQ and PR are tangents from P to a circle centre O as


shown in the figure above. If QRP = 340. Find the angle
marked x.
A. 340 B. 560 In the figure above, PQ//ST, RS//UV. If PQR = 350 and
0 0
C. 68 D. 112 QRS = 650, find STV
A. 300 B. 350
0
C. 55 D. 650
44. An open rectangular box externally measures 4m x 3m x 48. The people in a city with a population of 109 million were
4m. find the total cost of painting the box externally if it grouped according to their ages. Use the diagram below
costs #2.00 to paint onesquare metre. to determine the number of people in the 15-29 years
A. #96.00 B. #112.00 group.
C. #136.00 D. #160.00

45. Of the nine hundred students admitted in a university


in 1979, the following was the distribution by state 24
O

Anambra 185
O
Imo 135 52 O
Kaduna 90 116
O
Kwara 110 64
Ondo 155 104
O

Oyo 225
In a pie chart drawn to represent this distribution, the
angle subtended at the centre by Anambra is
A. 500 B. 650
0
C. 74 D. 880
A. 29 x 104 B. 26 x 104
46. Find the median of the numbers 89, 141, 130, 161, 120, C. 16 x 104 D. 13 x 104
131, 131, 100, 108 and 119
A. 131 B. 125 49. A man kept 6black, 5 brown and 7 purple shirts in a
C. 123 D. 120 drawer. What is the probability of his picking a purple
shirt with his eyes closed?
47. Find the probability that a number selected at random A. 1/7 B. 11/18
from 40 to 50 is a prime C. 7/18 D. 7/11
A. 3/11 B. 5/11
C. 3/10 D. 4/11 50. The table below gives the scores of a group of students
in a Mathematics test

If the mode is m and the number of students who scored


4 or less is S. What is (s, m)?
A. (27,4 ) B. (14, 4)
C. (13, 4) D. (4, 4)

Mathematics 2009

1. Convert 241 in base 5 to base 8 4. Reduce each number to two significant figures and then
A. 718 B. 1078 evaluate (0.02174 x 1.2047)
C. 1768 D. 2418 0.023789
A. 0.8 B. 0.9
2. Find the least length of a rod which can be cut into C. 1.1 D. 1.2
exactly equal strips, each of either 40cm or 48cm in
length. 5. A train moves from P to Q at an average speed of 90km/
A. 120cm B. 240ccm hr and immediately returns from O to P through the
C. 360cm D. 480cm same route and at an average speed of 45km/h. find the
average speed for the centre journey.
3. A rectangular has lawn has an area of 1815square yards. A. 55 00km/hr B. 60 00km/hr
If its length is 50meters, find its width in metres. Given C. 67.50km/hr D. 75 00km/hr
that 1meters equals 1.1yards
A. 39.93 B. 35.00 6. If the length of a square is increased by 20% while its
C. 33.00 D. 30.00 width is decreased by 20% to form a rectangle, what is
the ratio of the area of the rectangle to the area of the
square?
A. 6.5 B. 25.24
C. 5.6 D. 24.25
7. Two brothers invested a total of #5,000.00 on a farm 18. The formula Q = 15 + 0 5n gives the cost Q (in Naira) of
project. The farm yield was sold for # 15, 000.00 at the feeding n people for a week. Find in kobo the extra cost
end of the season. If the profit was shared in the ratio of feeding one additional person.
2:3, what is the difference in the amount of profit A. 350k B. 200k
received by the brothers? C. 150k D. 50k
A. #2,000.00 B. #4,000.00
C. #6,000.00 D. #10,000.00 19. If P varies inversely as V and V varies directly as R2,
find the relationship between P and R given that R = 7
8. Peter’s weekly wages are #20.00 for the first 20 weeks when P = 2
and #36.00 for the next 24 weeks. Find his average A. P = 98R2 B. PR2 = 98
weekly wage for the remaining 8 weeks of the year. If C. P = 1/98R D. P = R2/98
his average weekly wage for the whole year is #30.00
A. #37.00 B. #35.00 20. Make y the subject of the formula
C. #30.00 D. #5.00 Z = x2 + 1/y3
A. y = 1 B. y= 1
9. A man invests a sum of money at 4% per annum simple (z - x2) 3 (Z + x3) 1/3
interest. After 3 years, the principal amounts to
#7,000.00. find the sum invested C. y= 1 D. y = 1
A. #7,840.00 B. #6,250.00 (Z - x2) 1/ 3 3
  3 x2
C. #6,160.00 D. #5,833.33
21. Find the values of m which make the following quadratic
10. By selling 20 oranges for #1.35 a trader makes a profit function a perfect square
8%. What is his percentage gain or loss if he sells the x2 + 2 (m + 1) x + m + 3
same 20oranges for #1.10? A. -1, 1 B. –1, 2
A. 8% B. 10% C. 1, -2 D. 2, -2
C. 12% D. 15%
22. Factorize 62x+ 1 + 7(6x) - 5
11. Four boys and ten girls can cut a field in 5 hours. If the A. {3(6x) – 5} {2(6x)} + 1}
boys work at 1/4 the rate of which the girls work, how B. {3(6x) – 5} {2(6x)} - 1}
many boys will be needed to cut the field in 3 hours? C. {2(6x) – 5} {3(6x)} + 1}
A. 180 B. 60 D. {2(6x) – 5} {3(6x)} - 1}
C. 25 D. 20
23. Find two values of y which satisfy the simultaneous
12. Evaluate without using tables. equations x + y = 5, x2 – 2y2 = 1
A. 625/8 B. 8/625 A. 12, -2 B. –12, 12
C. 1/8 D. 8 C. –12, 2 D. 2, -2

13. Instead of writing 35/6 as a decimal correct to 3 24. An (n - 2)2 sided figure has n diagonals find the number
significant figures, a student wrote it correct to 3 places n ofdiagonals for a 25 sided figure
of decimals. Find his error in standard form A. 7 B. 8
A. 0.003 B. 3.0 x 10-3 C. 9 D. 10
2
C. 0.3 x 10 D. 0.3 x 10-3
25.
14. Simplifywithout using tables
(Log26 – Log23)/(Log28- 2Log21/2) f(x)
A. 1/5 B. ½
C. –1/2 D. Log23/Log27

15. Simplify without using tables


2 14 x 321) / 724x 298)
A. 3 B. 3 -1 0 1
4 4
C. 3 14 D. 3 2
28 28 A cubic function f(x) is specified by the graph show
above. The values of the independent variable for which
16. If p – 2/3 (1 – r2)/n2, find n when r = Ö1/3 and p = 1
A. 3/2 B. 3 the function vanishes are
A. -1, 0, 1 B. –1 < x < 1
C. 1/3 D. 2/3
C. x, - 1 D. x>1

17. If a =U2–3V2 and b = 2UV + V2 evaluate (2a - b) (a – b3 ),


26. Solve the inequality x – 1 > 4(x + 2)
when u = 1 and v = -1
A. x> -3 B. x< -3
A. 9 B. 15
C. 2 < x < 3 D. –3 < x < -2
C. 27 D. 33
27. Simplify(x2 - y2) / (2x2 + xy-y2) Y
A. x+-y B. x +y 34.
2x + y 2x - y
C. x - y D. x - y
2x - y 2x + y

28. The minimum value of y in the equation


y = x2 – 6x + 8 is 9 cm 16 cm
X Z
A. 8 B. 3 T
C. 0 D. –1

29. Find the sum of the first 21 terms of the progression – In the figure above, XYZ = YTZ = 900, XT = 9cm and
10, -8,-6,…. TZ = 16cm. Find YZ
A. 180 B. 190 A. 25cm B. 20cm
C. 200 D. 210 C. 16cm D. 9cm

30. Find the eleventh term of the progression 4, 8, 16,.. 35. Two chords QR and NP of a circle intersect inside the
A. 213 B. 212 circle at X. if RQP = 370, RQN = 490 and QPN = 350, find
11
C. 2 D. 210 PRQ
A. 350 B. 370
31. Q C. 49 0
D. 590
x
36.
R
T y
O 30
O
y

x
P
x
x

In the diagram above, POQ is a diameter, O is the centre


of the circle and TP is a tangent. Find the value of x.
A. B. 400 In the figure above, find the value of x.
C. 450 D. 500 A. 1100 B. 1000
C. 900 D. 800
32. P
2h
37. Q R
2 cm

3h
Q R

P S
T S 2 cm 2 cm

In the diagram above, QR//TS, QR:TS = 2:3. find the In the figure above, PQRS is a rectangle. If the shaded
ratio of the area of triangle PQR to the area of the area is 72sq.cm find h
trapezium QRST A. 12cm B. 10cm
A. 4:9 B. 4:5 C. 8cm D. 5cm
C. 1:3 D. 2:3
38. The sine, cosine and tangent of 2100 are respectively
33. Three angle s of a nonagon are equal and the sum of six
other angles is 11100. Calculate the size of one of the A. -1/2, 3/2, 3/3 B. 1/2, 3/2 3/3
equal triangles
A. 2100 B. 1500 C. 3/2, 3/3, 1 D. 3/2, 1/2 1
0
C. 105 D. 500
39. If tan q = (m2 – n2)/2mn, find sec q
A. (m2 + n2)/(m2 – n2) B. (m2+n2)/2mn
C. mn/2(m2–n2) D. m2 n2/(m2 – n2)
45. What is the locus of the mid-points of all chords of length
40. From two points X and Y, 8m apart, and in line with a pole, 6cm within a circle of radius 5cm and with centre O.
the angle of elevation of the top of the pole are 300 and A. A circle of radius 4cm and with centre O
600 respectively. Find the height of the pole, assuming B. The perpendicular bisector of the chords
that X, Y and the foot of the pole are on the same C. A straight line passing through center O
horizontal plane. D. A circle of radius 6cm and with centre O
A. 4m B. 83/2m
C. 43m D. 83m 46. Taking the period of daylight on a certain day to be
from 5.30a.m to 7.00p.m, calculate the period of daylight
41. A room is 12m long. 9m wide and 8m high. Find the and of darkness on that day
cosine of the angle which a diagonal of the room makes A. 187030’172030’ B. 1350225’
0 0
with the floor of theroom C. 202 30’157 30’ D. 1950165’
A. 15/17 B. 8/17
C. 8/15 D. 12/17 47. The goals scored by 40 football teams from three league
divisions are recorded below
42. What is the circumference of radius of the earth?
A. R cos q B. 2p R cos q
C. R sin q D. 2p R sin q
What is the total number of goals scored by all the
43. The base of a pyramid is a square of side 8cm. If its teams?
vertex is directly above the centre, find the height, given A. 21 B. 40
that the edge is 4.3cm C. 91 D. 96
A. 6cm B. 5cm
C. 4cm D. 3cm 48. The numbers 3,2,8,5,7,12,9 and 14 are the marks scored
P by a group by a group of students in a class test if P is
44. the mean and Q the median the P + Q is
A. 18 B. 171/2
C. 16 D. 15

49. Below are the scores of a group of students in a music


test

Q R
If CF(x) is the number of students with scores less than
or equal to x, find CF(6)
A. 40 B. 38
C. 33 D. 5

50. Find the probability of selecting a figure which is


X parallelogram from a square, a rectangle, a rhombus, a
kite and a trapezium
The figure above is an example of the construction of a A. 3/5 B. 2/5
A. perpendicular bisector to a given straight line C. 4/5 D. 1/5
B. perpendicularfrom a givenpointtoa givenline
C. perpendicular to a line from a given point on
that line
D. given angle.

Mathematics 1988
1. Simplify (1 1 / (2 1 of32)
2 4 3. A 5.0g of salts was weighed by Tunde as 5.1g. what is
A. 3/256 B. 3/32 the percentage error?
C. 6 D. 85 A. 20 B. 2
C. 2 D. 0.2
2. If x is the addition of the prime numbers between 1 and
6, and y the H. C.F of 6,9, 15, find the product of x and 4. Find correct to one decimal place,
y 0.24633 /0.0306
A. 27 B. 30 A. 0.8 B. 1.8
C. 33 D. 90
C. 8.0 D. 8.1
2010 JAMB MATHEMATICS QUESTIONS

PAPER TYPE: B percentage error in the


measurement?
1. Which Mathematics Question
Paper Type is given to you? A. 0.40%
B. 0.01%
A. Type A C. 0.25%
B. Type B D. 0.80%
C. Type C
D. Type D 5. At what rate will be interest on
₦400 increase to ₦ 24 in 3 years
2. Find r, if 6r78 =5119 reckoning in simple interest?

A. 5 A. 4%
B. 2 B. 2%
C. s C. 3%
D. 6 D. 5%

7 1
3. Simplify (3 of ÷9 ) 6. If p: q = 2
: 5 and q: r = 3 : 1,
4 9 2 3 6 4 2

find 𝑝: 𝑞: 𝑟.
A. 1/25
B. 1/4 A. 9:10:15
C. 1/5 B. 12:15:16
D. 1/36 C. 12:15:10
D. 10:15:24
4. A student measures a piece of
rope and found that it was 1.26 m 1
−4
7. Evaluate(81 ) x 2−1
16
long. If the actual length of the
rope was 1.25 m, what was the

pdfmadeazy.com.ng
A. 1 11. Which of the Venn diagrams
3

B. 6 below represents P' ∩ Q' ∩ R' ?

C. 3
D. 1
6

8. Given that log 2 = 0.3010, log


7 = 0.8451. Evaluate log 112.

A. 2.5441 12. In a survey of 50 newspaper


B. 2.0491 readers, 40 read Champion and
C. 2.1461 30 read Guardian, how many read
D. 3.1461 both papers?

9. Rationalise 2√3+√5 A. 15
√5−√3 B. 5
C. 10
3√15+11
A. D. 20
2
3√15−11
B.
2
13. Make Q the subject of formula
C. 3√15 − 11
if P M(X + Q)+ 1
D. 3√15 − 11 5

5P+MX−5
10. Express the product of 0.21 A. M
5P−MX−5
and 0.34 in standard form B. M
5P−MX+5
C. M
5P+MX+5
A. 7.14 x 10-3 D. M

B. 7.14 x 10-1
C. 7.14 x 10-2

D. 7.14 x 10-4

pdfmadeazy.com.ng
14. If 9x2 + 6xy + 4y2 is a factor A. 5
of 27x3 - 8y3, find the other B. 12
factor. C. 6
D. 3
A. 3x - 2y
B. 2y - 3x 18. If x is inversely proportional
1
C. 2y + 3x to y and x = 2 when y = 2, find
2
D. 3x + 2y x if y = 4

2+3x2−10x
15. Factorize completely x
A. 21
2x2−8 4

B. 5
x2+5
A. 2x+4 C. 4
x(x+5)
B. 2(x+2) D. 11
4
x(x−5)
C. 2(x+2)

19. For what range of values of x


x(x−5)
D. is 1 𝑥 + 1 > 1 x + 1
2(x−2)
2 4 3 2

16. Solve for x and y if x - y = 2 A. x > −3


2
and x2 - y2 = 8
B. x >3
2

A. (1, 3) C. x <2
3
2
B. (3, 1) D. x > −
3
C. (-1, 3)
D. (-3, 1)
20. Solve the inequalities −6 ≤ 4
— 2𝑥 < 5 − 𝑥
17. If y varies directly as the
square root of x and y =3 when x
A. -1 ≤ x < 6
= 16, calculate y when x = 64
B. -1 < x ≤ 5

pdfmadeazy.com.ng
C. -1 < x < 5 D. 55
D. -1 ≤ x ≤ 6
24. If p and q are two non-zero
21. Find the sum to infinity of the numbers and 18(𝑝 + 𝑞) = (18 +
following series. 0.5 + 0.05 + 𝑝)𝑞, which of the following must
0.005 + 0.0005 + ..... be true?

A. 5 A. q = 18
9

B. 5 B. p = 18
7
C. p <1
C. 5
8 D. q < 1
D. 5
11
x 3
25. If │ │ = 15, find the value
2 7
22. The 3rd term of an arithmetic
of x
progression is -9 and the 7th term
is -29. Find the 10th term of the
A. 3
progression
B. 5
C. 4
A. 44
D. 2
B. -165
C. -44 2
D. 165 26. Evaluate |4 6 3|
8 9 1

23. If x * y = x + y2, find then


A. -42
value of (2*3)*5
B. 102
C. 18
A. 36
D. -102
B. 11
C. 25

pdfmadeazy.com.ng
27. If P = [
2 −3
], what is P-1 45)⁰ and (x + 10)⁰. Find the
1 1
value of the least interior angle.

1 3

A. [15 A. 102⁰
2]
5

5 5 B. 52°
1 3
C. 82⁰
B. [ 5 5]
1 2
−5 D. 112°
5

1 3

C. [ 5 5
1 2]
−5 5

1 3
− −5
D. [ 5 ]
1 2
−5 −5

30. From the cyclic quadrilateral


TUVW above, find the value of x

28. From the diagram above, find


A. 23°
x
B. 26°
C. 24°
A. 65°
D. 20°
B. 50°
C. 55°
31. If the two smaller sides of a
D. 75°
right-angled triangle are 4 cm and
5 cm, find its area.
29. The interior angles of a
quadrilateral are (x + 15)⁰, (2x -
A. 10 cm2

pdfmadeazy.com.ng
B. 6 cm2 A. y = -x + 6
C. 8 cm2 B. y = -x + 6
D. 24 cm2 C. y = -x - 6
D. y = x - 6
32. An arc subtends an angle of
50° at the centre of circle of 35. Find the distance between the
radius 6cm. Calculate the area of points (1 , 1) and (− 2 , − 2)
1 1
2 2
the sector formed

A. √2
90
A. cm2
7
0
B.
110
B. cm2 C. 1
7

C.
100 D. √3
7
cm2
80
D. cm2
7
36. Find the gradient of the line
passing through the points P(1, 1)
33. A cylindrical pipe 50cm long
and Q(2, 5).
with radius 7m has one end open.
What is the total surface area of
A. 4
the pipe?
B. 2
C. 3
A. 700 πm 2
D. 5
2
B. 98 πm
C. 350 πm2
37. Find the equation of a line
D. 749 πm 2
parallel to y = -4x + 2 passing
through (2,3)
34. What is the locus of point that
is equidistant from points P(1,3)
A. y - 4x + 11 = 0
and Q(3,5)?
B. y - 4x - 11 = 0
C. y + 4x + 11 = 0

pdfmadeazy.com.ng
D. y + 4x - 11 = 0 A. 3(2x+1)2
B. 3(2x + 1)
38. If cot θ = 8 , where θ is acute, C. 6(2x + 1)
15

find sin θ D. 6(2x+1)2

A.
13 41. If y = xsinx, find dy
15 dx
15
B. 17

8 A. 𝑠i𝑛 𝑥 + 𝑥 𝑐o𝑠𝑥
C. 17

D.
16
17
B. 𝑠i𝑛𝑥 + 𝑥 𝑐o𝑠𝑥
C. 𝑠i𝑛𝑥 − 𝑥𝑐o𝑠𝑥
D. 𝑠i𝑛𝑥 – 𝑐o𝑠𝑥

42. At what value of X does the


function 𝑦 = −3 − 2x + x2 attain a
minimum value?

A. 1
B. -4
39. If the area of 𝗈 PQR above is C. -1
12√3 cm2, find the value of q? D. 4

A. 6 cm 2
43. Evaluate ∫0 (x3 + x 2) dx
B. 8 cm
C. 7 cm
A. 25
6
D. 5 cm
B. 62
3

40. If y = (2x + 1)3, find dy C. 45


dx 6

D. 125
6

pdfmadeazy.com.ng
44. Find ∫ (sin x + 2) dx B. 20
C. 16
A. cos x + x2 + k D. 13
B. cos x + 2x + k
C. –cos x + 2x + k 47. Find the mean mark
D. –cos x + x2 + k
A. 3.2
45. B. 3.0
Marks 2 3 4 5 6 7 8 C. 3.1
Students 3 1 5 2 4 2 3
D. 3.3

From the table above, if the pass


48. Find the standard deviation of
mark is 5, how many students
2, 3, 5 and 6
failed the test?

5
A. 7 A. √
2

B. 2
B. √10
C. 6
C. √6
D. 9
D. √2
5

Use the table below to answer


question 46 and 47 49. In how many ways can a
Marks 1 2 3 4 committee of 2 women and 3 men
Frequency 2 2 8 4
be chosen from 6 men and 5
women?
46. The table above shows the
marks obtained in a given test.
A. 50
How many students failed the
B. 200
test?
C. 100
D. 30
A. 15
pdfmadeazy.com.ng
50. If three unbiased coins are Facebook Study Group
tossed, find the probability that
they are all heads. If you need an explanation to
any of the answers or you want to
A. 1 download more past questions
8

B. 1 for FREE.
3

C. 1
6 Click to ENTER our Facebook
D. 1
study group
9

ANSWERS

1. B 2. C 3. D 4. D 5. B 6. C 7. A
8. B 9. A 10. C 11. A 12. D 13. B
14. A 15. A 16. B 17. C 18. D
19. B 20. B 21. A 22. C 23. A
24. A 25. A

26. D 27. B 28. A 29. B 30. C


31. A 32. B 33. D 34. A 35. A
36. A 37. D 38. B 39. A 40. D
41. B 42. C 43. B 44. C 45. D
46. B 47. D 48. A 49. B 50. A

pdfmadeazy.com.ng
2011 JAMB MATHEMATICS QUESTIONS

TYPE: D A. ₦150
B. ₦220
1. Which Mathematics Question C. ₦130
Paper Type is given to you? D. ₦250

A. Type A 5. If the numbers M, N, Q are in


B. Type B the ratio 5:4:3, find the value of
C. Type C 2N−Q
M
D. Type D

A. 2
2. If 2q35 = 778, find q B. 3
C. 1
A. 2 D. 4
B. 1
C. 4 1 9 1
6. Simplify 16 4 ÷ ( ) −2
(81)
D. 0 16

2 5
33 × 6 × 3
2
A. 23
3. Simplify 11 3 2
15 × 4 × 27 B. 12
C. 89

A. 52 D. 13
3

B. 30
7. If log3 18 + log3 3 - log3 x = 3,
C. 41
3 find x.
D. 50

A. 1
4. A man invested ₦5,000 for 9
B. 2
months at 4%. What is the simple
C. 0
interest?
pdfmadeazy.com.ng
D. 3 B. {b, d}
C. {a, e}
8. Rationalize 2−√5 D. {c}
3−√5

1−√5
11. Raial has 7 different posters
A.
2 to be hanged in her bedroom,
1−√5
B. living room and kitchen.
4

C. √5−1 Assuming she has plans to place


2
1+√5 at least a poster in each of the 3
D.
4
rooms, how many choices does

1 1 she have?
9. Simplify [√2 + ] [√2 − ]
√3 √3
A. 49

A. 7 B. 170
3
C. 21
B. 5
3 D. 210
C. 5
2

D. 3 12. Make R the subject of the


2
2+M
formula if T = KR
3

3T−K
A. √
M

3T+M
B. √
K

3T+K
C. √
10. From the Venn diagram M

above, the complement of the set 3T−K


D. √
K
P ∩ Q is given by

A. {a, b, d, e}

pdfmadeazy.com.ng
13. Find the remainder when 𝑥3 – A. 201
4
2𝑥2 + 3𝑥 − 3 is divided by 𝑥2 + 1 B. 27
C. 21
4
A. 2x - 1
D. 36
B. x + 3
C. 2x + 1
17. T varies inversely as the cube
D. x – 3
of R. When R = 3, T = 2, find T
81

14. Factorize completely 9y2 – when R = 2

16x2
A. 1
18

A. (3𝑦 − 2𝑥)(3𝑦 + 4𝑥) B. 1


12
B. (3𝑦 + 4𝑥)(3𝑦 + 4𝑥) C. 1
24
C. (3𝑦 + 2𝑥)(3𝑦 − 4𝑥)
D. 1

D. (3𝑦 − 4𝑥)(3𝑦 + 4𝑥) 6

18. Which of the following


15. Solve for x and y respectively
diagrams represents the solution
in the simultaneous equations -2x
of the inequalities y ≤ x – 2 and y
- 5y = 3, x + 3y = 0
≥ x2 – 4 ?

A. -3, -9
B. 9, -3
C. -9,3
D. 3, -9

16. If x varies directly as square


root of y and x = 81 when y = 9,
Find x when y = 17
9 19. Solve the inequality -6(x + 3)
≤ 4(x - 2)
pdfmadeazy.com.ng
A. x ≤ 2 D. 64
B. x ≥ -1
C. x ≥ -2 23. A binary operation ⊕ on real
D. x ≤ -1 numbers is defined by x ⊕ y = xy
+ x + y for two real numbers x
2
20. Solve the inequality 𝑥2 + 2𝑥 > and y. Find the value of 3 ⊕ −
3
15.
1
A. −
2
A. x < -3 or x > 5
B. 1
B. -5 < x < 3 3

C. x < 3 or x > 5 C. -1

D. x > 3 or x < -5 D. 2

2 3 4 1
21. Find the sum of the first 18 24. If │ │=│ │, find the
5 3x 1 2𝑥
terms of the series 3, 6, 9..., 36. value of x.

A. 505 A. -6
B. 513 B. 6
C. 433 C. -12
D. 635 D. 12

22. The second term of a 4 2 −1


geometric series is 4 while the 25. Evaluate | 2 3 −1|
−1 1 3
fourth term is 16. Find the sum of
the first five terms
A. 25
B. 45
A. 60
C. 15
B. 62
D. 55
C. 54

pdfmadeazy.com.ng
26. The inverse of matrix N = 29. A chord of circle of radius 7
[2 3] is cm is 5 cm from the centre of the
1 4
maximum possible area of the
square?
A. [2 1]
1
5 3 4
4 −3
B. 1 [ ] A. 4√6 cm2
5 −1 2
2 −1 B. 3√6 cm2
C. 1 [ ]
5 −3 4 C. 6√6 cm2
4 1
D. 1 [ ] D. 2√6 cm2
5 3 2

27. What is the size of each 30. A solid metal cube of side 3

interior angle of a 12-sided cm is placed in a rectangular tank

regular polygon? of dimension 3, 4 and 5 cm. What


volume of water can the tank now

A. 120° hold?

B. 150°
C. 30° A. 48 cm3

D. 180° B. 33 cm3
C. 60 cm3

28. A circle of perimeter 28cm is D. 27 cm3

opened to form a square. What is


the maximum possible area of the 31.The perpendicular bisector of a

square? line XY is the locus of a point

A. 56 cm2 A. whose distance from X is

B. 49 cm2 always twice its distance from Y

C. 98 cm2 B. whose distance from Y is

D. 28 cm2 always twice its distance from X


C. which moves on the line XY

pdfmadeazy.com.ng
D. which is equidistant from the 35. A man walks 100 m due West
points X and y from a point X to Y, he then walks
100 m due North to a point Z.
32. The midpoint of P(x, y) and Find the bearing of X from Z.
Q(8, 6). Find x and y. midpoint =
(5, 8) A. 195°
B. 135°
A. (2, 10) C. 225°
B. (2, 8) D. 045°
C. (2, 12)
D. (2, 6) 36. The derivatives of (2𝑥 + 1)(3𝑥
+ 1) is
33. Find the equation of a line
perpendicular to line 2𝑦 = 5𝑥 + 4 A. 12x + 1
which passes through (4, 2). B. 6x + 5
C. 6x + 1
A. 5𝑦 − 2𝑥 − 18 = 0 D. 12x + 5
B. 5𝑦 + 2𝑥 − 18 = 0
C. 5𝑦 − 2𝑥 + 18 = 0 37. Find the derivative of sin 𝜃
cos 𝜃
D. 5𝑦 + 2𝑥 − 2 = 0

A. 𝑠e𝑐2 𝜃
34. In a right-angled triangle, if B. 𝑡𝑎𝑛 𝜃𝑐o𝑠e𝑐 𝜃
tan θ = 34. What is 𝑐o𝑠𝜃 − 𝑠i𝑛𝜃?
C. 𝑐o𝑠e𝑐𝜃𝑠e𝑐 𝜃
D.𝑐o𝑠e𝑐2𝜃
A. 2
5

B. 3 38. Find the value of x at the


5

C. 1 minimum point of the curve 𝑦 = 𝑥3


5
+ 𝑥2 − 𝑥 + 1
D. 4
5

pdfmadeazy.com.ng
A. 1 41. The pie chart shows the
3

B. −1 distribution of courses offered by


3
students. What percentage of the
C. 1
students offer English?
D. 1

A. 30%
1
39. Evaluate ∫0 (3 − 2x) 𝑑𝑥
B. 25%
C. 35%
A. 3 D. 20%
B. 5
C. 2
D. 6

40. Find ∫ cos 4 𝑥𝑑𝑥

A. 3 sin 4𝑥 + 𝑘
4

B. −1 sin 4𝑥 + 𝑘
4

C. −3 sin 4𝑥 + 𝑘
4

D. 1 sin 4𝑥 + 𝑘
4

42. The bar chart above shows


the distribution of SS2 students in
a school. Find the total number of
students

A. 180
B. 135
C. 210
D. 105

pdfmadeazy.com.ng
43. The sum of four consecutive D. 7
integers is 34. Find the least of
these numbers Class 3-5 6-8 9-11
Interval
Frequency 2 2 2
A. 7
B. 6 46. Find the standard deviation of
C. 8 the above distribution.
D. 5
A. √5
Number 0 1 2 3 5 B. √3
Frequency 1 4 8 2 5
C. √7
D. √2
44. From the table above, find the
median and range of the data
47. In how many was can the
respectively.
letters of the word ELATION be
arranged?
A. (8,5)
B. (3, 5)
A. 6!
C. (5, 8)
B. 7!
D. (5, 3)
C. 5!
D. 8!
Class 0-2 3-5 6-8 9-11
Interval
Frequency 3 2 5 3
48. In how many ways can five
people sit round a circular table?
45. Find the mode of the above
distribution
A. 24
B. 60
A. 9
C. 12
B. 8
D. 120
C. 10
pdfmadeazy.com.ng
49. Find the probability that a 14. D 15. C 16. D 17. B 18. B 19.
number picked at random from B 20. B 21. B 22. B 23. B 24. A
the set (43, 44, 45, ..., 60) is a 25. A
prime number.
26. B 27. B 28. B 29. A 30. B 31.
A. 2 D 32. A 33. B 34. C 35. B 36. D
3

B. 1 37. A 38. B 39. C 40. A 41. B 42.


3
A 43. A 44. B 45. D 46. B 47. B
C. 2
9 48. B 49. C 50. A
D. 7
9

50. In a class of 60 students, 30


offer Physics and 40 offer
Chemistry. If a student is picked
at random from the class, what is
the profitability that the student
offers both Physics and
Chemistry?

A. 1
3

B. 1
4

C. 1
2

D. 1
6

ANSWERS

1. D 2. A 3. D 4. A 5. C 6. B 7. B
8. B 9. B 10. A 11. D 12. D 13. A

pdfmadeazy.com.ng
2012 JAMB MATHEMATICS QUESTIONS

TYPE: GREEN 4. Evaluate 21


to 3 significant
9

figures
1. Which Question Paper Type of
Mathematics as indicated above is A. 2.30
given to you?
B. 2.31
C. 2.32
A. Type Green D. 2.33
B. Type Purple
C. Type Red
5. A man earns ₦3,500 per month
D. Type Yellow out of which he spends 15% on
his children's education. If he
2. Convert 726 to a number in
spends additional ₦1,950 on food,
base three how much does he have left?

A. 2211 A. ₦525
B. 2121
B. ₦1,025
C. 1212
C. ₦1,950
D. 1122
D. ₦2,975

2 1
23 × 1 2
3. Simplify 6. If 27x + 2 ÷ 9x + 1 = 32x, find x
44
5

A. 3
A. 114
B. 4
B. 116
C. 5
C. 56
D. 6
D. 45

7. If log3 𝑥2 = -8, what is x?

pdfmadeazy.com.ng
A. 13 D. 4
B. 19
C. 127 11. Make 'n' the subject of the
D. 181 formula if w = v(2+cn
1−cn

8. Simplify (√6 + 2)2 - (√6 + 2)2 1 w−2v


A. ( )
c v+w
B. 1 w−2v
( )
A. 2√6 c v−w
B. 4√6 C. 1 w+2v
( )
c v−w
C. 8√6 D. 1 w+2v
( )
c v+w
D. 16√6

12. Find the remainder when 2x3 -


9. If P is a set of all prime factors
11x2 + 8x - 1 is divided by x + 3
of 30 and Q is a set of all factors
of 18 less than 10, find P ∩ Q
A. -871
B. -781
A. {3}
C. -187
B. {2,3}
D. -178
C. {2,3,5}
D. {1,2}
13. Solve for x and y in the
equations below
10. In a class of 46 students, 22
x2 - y2 = 4
play football and 26 play
x+y=2
volleyball. If 3 students play both
games, how many plays neither?
A. x = 0, y = -2
B. x = 0, y = 2
A. 1
C. x = 2, y = 0
B. 2
D. x = -2, y = 0
C. 3

pdfmadeazy.com.ng
14. If y varies directly as √𝑛 and 17. Find the range of values of m
y = 4 when n = 4, find y when n which satisfy (m -3)(m - 4) < 0
= 17
9
A. 2 < m < 5

A. √17 B. -3 < m < 4

B. 4 C. 3 < m < 4
3
D. -4 < m < 3
C. 8
3

D. 2
3

15. U is inversely proportional to


the cube of V and U = 81 when V
= 2. Find U when V = 3

A. 24
B. 27 18. The shaded region above is
C. 32 represented by the equation
D. 36
A. y ≤ 4x + 2
16. The value of y for which B. y ≥ 4x + 2
1 1
1y + < y + 2 is C. y ≤ -4x + 4
5 5 2 5
D. y ≤ 4x + 4

A. y>2
3
19. The nth term of a sequence is
B. y<2
3 n2 - 6n - 4. Find the sum of the
2
C. y>−
3 3rd and 4th terms.
2
D. y<−
3
A. 24
B. 23

pdfmadeazy.com.ng
C. -24 A. −45
D. -25 B. −25
C. 4
20. The sum to infinity of a D. 5
geometric progression is − 1 and
10
the first term is −1. Find the 23. If │5 3│ = │3 5│, find the
8 x 2 4 5
common ratio of the progression. value of x

A. −1 A. 3
5
B. 4
B. −1
4
C. 5
C. −1
3 D. 7
D. −1
2
24. Given that 13 is a unit matrix

21. The binary operation * is of order 3, find |13|

defined on the set of integers


such that p * q = pq + p - q. Find A. -1

2 * (3 * 4) B. 0
C. 1

A. 11 D. 2

B. 13
C. 15
D. 22

22. The binary operation on the


set of real numbers is defined by
m*n = mn2 for all m, n ∈ R. If the
identity element is 2, find the
inverse of -5
pdfmadeazy.com.ng
25. In diagram above, QR//TU, B. (90 – 𝑥)°
< PQR = 80o and < PSU= 95o. C. (90 + 𝑥)°
Calculate < SUT. D. (180 – x)°

A. 15⁰
B. 25⁰
C. 30⁰
D. 80⁰

26. The angles of a polygon are


given by x, 2x, 3x, 4x and 5x 28. Find the area of the trapezium
respectively. Find the value of x. above.

A. 24⁰ A. 91 cm2
B. 30⁰ B. 78 cm2
C. 33⁰ C. 60 cm2
D. 36⁰ D. 19 cm2

29. A circular arc subtends angle


150° at the centre of a circle of
radius 12 cm. Calculate the area
of the sector of the arc?

A. 30 π cm2
27. In the diagram above, PQR is
B. 60 π cm2
a circle centre O. If < QPR is XO,
C. 120 π cm2
find < QRP.
D. 150 π cm2

A. X⁰

pdfmadeazy.com.ng
30. Calculate the volume of a 33. The distance between the
cuboid of length 0.76 cm, breadth point (4, 3) and the intersection
2.6 cm and height 0.82 cm. of 𝑦 = 2𝑥 + 4 𝑎𝑛𝑑 𝑦 = 7 − 𝑥 is

A. 3.92 cm3 A. √13


B. 2.13 cm3 B. 3√2
C. 1.97 cm3 C. √26
D. 1.62 cm3 D. 10√5

31. The locus of a point 34. Find the equation of the line
equidistant from the intersection through the points (-2, 1) and
1
of lines 3𝑥 − 7𝑦 + 7 = 0 𝑎𝑛𝑑 4𝑥 − (− , 4)
2
6𝑦 + 1 = 0 is a

A. y = 2x - 3
A. line parallel to 7𝑥 + 13𝑦 + 8= 0 B. y = 2x + 5
B. circle C. y = 3x - 2
C. semicircle D. y = 2x + 1
D. bisector of the line 7𝑥 + 13𝑦 +
8=0 35. If angle θ is 135⁰, evaluate
cos θ
32. The gradient of the straight
line joining the points P(5, -7) A. 1
2
and Q(-2, -3) is
B. √2
2

A. 12 C. −√2
2
B. 25 D. −1
2
C. −47
D. −23
36. A man stands on a tree
150cm high and sees a boat at an

pdfmadeazy.com.ng
angle of depression of 74°. Find C. − 3
11
the distance of the boat from the D. −
11
3
base of the tree.

A. 52 cm 40. Evaluate ∫04 sec2 𝜃𝑑𝜃

B. 43 cm
C. 40 cm A. 1

D. 15 cm B. 2
C. 3

37. If y = x2 - 1, find dy D. 4
x dx

A. 2x - 1
x2

B. 2x + x2
C. 2x - x2
D. 2x + 1
x2

38. Find dy if 𝑦 = cos 𝑥


dx 41. The grades of 36 students in a
class test are as shown in the pie
A. sin 𝑥 chart above. How many students
B. -sin 𝑥 have excellent?
C. tan 𝑥
D. -tan 𝑥 A. 12
B. 9
2
39. Evaluate ∫1 (𝑥2 − 4𝑥)𝑑𝑥 C. 8
D. 7
11
A. 3

3
B. 11

pdfmadeazy.com.ng
A. 9
B. 8
C. 7
D. 4

45. Find the range of 4, 9, 6, 3, 2,


42. The bar chart above shows
8, 10 and 11
the distribution of marks in a
class test. If the pass mark is 5,
A. 11
what percentage of students
B. 9
failed the test?
C. 8
D. 4
A. 10%
B. 20%
46. Find the standard deviation of
C. 50%
2, 3, 8, 10 and 12
D. 60%

A. 3.9
43. The mean of seven numbers
B. 4.9
is 96. If an eighth number is
C. 5.9
added, the mean becomes 112.
D. 6.9
Find the eighth number.

n+1
47. Evaluate Cn-2 If n =15
A. 126
B. 180
A. 3630
C. 216
B. 3360
D. 224
C. 1120
D. 560
44. Find the median of 2, 3, 7, 3,
4, 5, 8, 9, 9, 4, 5, 3, 4, 2, 4 and 5

pdfmadeazy.com.ng
48. In how many ways can the B. 3
15
letters of the word TOTALITY be C. 7
15
arranged?
D. 13
15

A. 6720
ANSWERS
B. 6270
C. 6207
1. A 2. D 3. D 4. D 5. B 6. B 7. D
D. 6027
8. C 9. B 10. A 11. A 12. D 13. C
14. C 15. A 16. C 17. B 18. C
49. The probability that a student
19. D 20. B 21. B 22. A 23. C
passes a physics test is 2. If he
3 24. C 25. A
takes three physics tests, what is
the probability that he passes two
26. D 27. B 28. C 29. B 30. D
of the tests?
31. B 32. C 33. B 34. B 35. C
36. B 37. D 38. B 39. D 40. A
A. 4
41. D 42. C 43. D 44. D 45. B
9

B. 6 46. A 47. D 48. A 49. D 50. D


9

C. 4
27

D. 2
27

50. The probabilities that a man


and his wife live for 80 years are 2
3

and 3
respectively. Find the
5

probability that at least one of


them will live up to 80 years

A. 2
15

pdfmadeazy.com.ng
2013 JAMB MATHEMATICS QUESTIONS

TYPE: U 4. Evaluate 1.25 × 0.025, correct to 1


0.05

decimal place
1. Which Mathematics Question
Paper Type is given to you? A. 6.3
B. 0.5
A. Type D
C. 0.6
B. Type I
D. 6.2
C. Type B
D. Type U 5. Calculate the time taken for
₦3000 to earn ₦600 if invested at
2. Convert 2710 to another 8% simple interest
number in base three

A. 31 years
2
A. 11003
B. 11 years
B. 10003 2
1
C. 10013 C.2 years
2

D. 10103 D. 3 years

3. 3 girls share a number of 6. Simplify 3 − 5𝑛91 − 𝑛 × 27𝑛+1


apples in the ration 5:3:2. If the
highest share is 40 apples, find A. 32
the smallest share B. 33
C. 35
A. 16 D. 3
B. 38
C. 36 7. If log10 4 = 0.6021, evaluate
D. 24
1
log10 4 3

pdfmadeazy.com.ng
A. 1.8063 C. {3, 5, 7, 11, 17, 19}
B. 0.2007 D. {3, 5, 11, 13, 17, 19}
C. 0.3011
D. 0.9021 11. If S = √t2 − 4t + 4 find t in
terms of S
√5(√147−√12
8. Simplify
√15
A. S - 2
B. S2 + 2
A. 1
9
C. S2 - 2
B. 9
D. S + 2
C. 5
D. 1
5 12. If 𝑥 − 4 is a factor of 𝑥2 − 𝑥 −
𝑘, then k is
9. P, Q and R are subsets of the
universal set U. The Venn A. 20
diagram showing the relationship B. 2
(P∩Q)𝖴R is C. 4
D. 12

13. The remainder when 6𝑝3 − 𝑝2


— 47𝑝 + 30 is divided by 𝑝 − 3 is

A. 63

10. if 𝑃 = {𝑥: 𝑥 i𝑠 o𝑑𝑑, −1 < 𝑥 ≤ B. 18

20} and 𝑄 i𝑠 {𝑦: 𝑦 i𝑠 𝑝𝑟i𝑚e, −2 < C. 21

𝑦 ≤ 25, find P ∩ Q D. 42

A. {3, 5, 7, 11, 13, 17, 19} 14. P varies jointly as m and u,

B. {2, 3, 5, 7, 11, 13, 17, 19} and varies inversely as q. Given

pdfmadeazy.com.ng
that p = 4, m = 3 and u = 2 and 17. The graph above is correctly
q = 1, find the value of p when m represented by
= 6, u = 4 and q = 8
5
A. 𝑦 = 𝑥2 − 𝑥 − 1

A. 10 B. 𝑦 = 𝑥2 + 𝑥 − 2
288 C. 𝑦 = 𝑥2 − 𝑥 − 2
B.
5
128
D. 𝑦 = 𝑥2 − 3𝑥 + 2
C.
5

D. 15
18. Solve for 𝑥: |𝑥 − 2| < 3

15. If r varies inversely as the


A. −1 < 𝑥 < 5
square root of s and t, how does s
B. 𝑥 < 1
vary with r and t?
C. 𝑥 < 5
D. −1 < 𝑥 < 3
A. s varies directly as r2 and t2
B. s varies directly as r and t
19. If the sum of the first two
C. s varies inversely as r and t2
terms of a G.P. is 3, and the sum
D. s varies inversely as r2 and t
of the second and the third terms
is -6, find the sum of the first
16. Evaluate 3(x + 2) > 6(x + 3)
term and the common ratio

A. x < -4
A. -5
B. x > 4
B. 5
C. x < 4
C. -2
D. x > -4
D. -3

20. The nth term of the


7 10 13
progression 4 , , , , … is
2 3 4 5

pdfmadeazy.com.ng
3n+1
A. n−1 2
3
A. [ 2 ]
5
B.
3n+1
n+1
−3 −
2
1−3n
C. n+1 3
2
B. [ 2
5]

D.
3n+1 −3
2
n+1
3
2 −
C. [ 2
5]
21. If a binary operation * is −3 −
2
defined by x * y = x + 2y, find 2 2 −
3

* (3 * 4) D. [ 5
2
]
−3
2

A. 14
B. 26
C. 24
D. 16

5 3 4 2
22. If P = [ ] and Q = [ ] 24. In the diagram above, find
2 1 3 5
the value of x.
find 2P + Q

7 7 A.45°
A. [ ]
8 14 B.15°
8 14 C.30°
B. [ ]
7 7
C. [
7 7
] D. 40°
14 8
14 8
D. [ ]
7 7

23. Find the inverse [5 3]


6 4

25. The value of x in the figure


above is

pdfmadeazy.com.ng
A. 70° A. 8√3 cm
B. 130° B. 4 cm
C. 110° C. 8 cm
D. 100° D. 8√2 cm

26. If the angles of a quadrilateral 29. A chord of a circle subtends


are (3y+10)°, (2y+30)°, (y+20)° an angle of 120° at the centre of
and 4y°, find the value of y. a circle of diameter 4√3cm.
Calculate the are of the major
A. 66° sector
B. 12°
C. 30° A. 32π cm2
D. 42° B. 4π cm2
C. 8π cm2
27. A square tile has side 30cm. D. 16π cm2
How many of these tiles will cover
a rectangular floor of length 7.2m 30. The locus of the points which
and width 4.2m? is equidistant from the line PQ
forms a
A. 720
B. 336 A. perpendicular line to PQ
C. 420 B. circle centre P
D. 576 C. circle centre Q
D. pair of parallel lines to PQ
28. Find the length of a chord
which subtends an angle of 90° at 31. If the mid-point of the line PQ
the centre of a circle whose radius is (2,3) and the point P is (-2,1).
is 8cm. find the coordinate of the point Q.

pdfmadeazy.com.ng
A. (8 ,6) D. 12
5
B. (5, 6)
C. (0, 4)
35. If y = (2x +2)3, find dy
dx
D. (6, 5)

A. 3(2𝑥 + 2)
32. Find the equation of the
B. 6(2𝑥 + 2)2
perpendicular bisector of the line
C. 3(2𝑥 + 2)2
joining P (2, 3) to Q(-5, 1)
D. 6(2x+2)

A. 8𝑦 + 14𝑥 + 13 = 0
36. if y = xsin 𝑥, find dy
B. 8𝑦 − 14𝑥 + 13 = 0 dx

C. 8𝑦 − 14𝑥 − 13 = 0
D. 8𝑦 + 14𝑥 − 13 = 0 A. cos 𝑥 + xsin 𝑥
B. sin 𝑥 + xcos 𝑥

33. In triangle PQR, q= 8cm, C. sin 𝑥 − cos 𝑥

r=6cm and cos p = 1 D. cos 𝑥− xsin 𝑥


12

37. The radius of a circle is


A. √108 cm
increasing at the rate of 0.02cms-
B. √9 cm
1
. Find the rate at which the area
C. √92 cm
is increasing when the radius of
D. 10 cm
the circle is 7cm.

34. If tan θ = 3, find the value of


4
A. 0.35 cm2S-1
sin 𝜃 + cos 𝜃
B. 0.88cm2S-1
C. 0.75 cm2S-1
A. 11
3 D. 0.55 cm2S-1
B. 12
3

C. 13 38. Integrate 1+x 𝑑𝑥


x3
5

pdfmadeazy.com.ng
A. 2x2 − 1 + k D.720 mins
x
1
B. − 1 − +k
2x2 x
2
C. −x — 1 + k
2 x

D. x2− 1 +k
x

39. Evaluate ∫02 sin 𝑥 𝑑𝑥

A. -2
B. 2 41. The pie chart above shows the
C. 1 statistical distribution of 80
D. -1 students in five subjects in an
examination. Calculate how many
student offer Mathematics.

A. 50
B. 20
C. 30
D. 40
40. The bar chart above shows
the allotment of time (in minutes)
42. Find the mean of t + 2, 2t - 4,
per week for selected subjects in
3t + 2 and 2t.
a certain school. What is the total
time allocated to the six subjects
A. 2t + 1
per week?
B. t
C. t + 1
A. 960 mins
D. 2t
B. 200 mins
C. 460 mins

pdfmadeazy.com.ng
43. The mean of seven numbers C.√3
is 10. If six of the numbers are 2, D. 2
4, 8, 14, 16 and 18, find the
mode. Score 3 4 5 6 7 8 9 10
Freq. 1 0 7 5 2 3 1 1

A. 14
46. The table above shows the
B. 2
scores of 20 students in further
C. 6
mathematics test. What is the
D. 8
range of the distribution?

A. 3
Age 20 25 30 35 40 45
B. 10
No. of 3 5 1 1 2 5
people C. 7
D. 6
44. Calculate the median age of
the frequency distribution in the
47. In how many ways can a
table above
student select 2 subjects from 5
subjects?
A. 35
B. 20 5!
A.
2!3!
C. 25
5!
B. 2!
D. 30
5!
C. 3!
5!
D. 2!2!
45. If the variance of 3 + 𝑥, 6, 4,
𝑥 𝑎𝑛𝑑 7 − 𝑥 i𝑠 4 and the mean is
48. In how many ways can 3
5, find the standard deviation
seats be occupied if 5 people are
willing to sit?
A. 3
B. √2

pdfmadeazy.com.ng
A. 5 ANSWERS
B. 120
C. 60 1. D 2. B 3. A 4. C 5. C 6. B 7. B
D. 20 8. C 9. C 10. B 11. B 12. D 13. D
14. A 15. D 16. A 17. B 18. C
49. What is the probability that an 19. A 20. D 21. C 22. D 23. D
integer x (1≤x≤25) chosen at 24. D 25. B
random is divisible by both 2 and
3? 26. B 27. B 28. D 29. B 30. A
31. D 32. C 33. C 34. D 35. B

A. 4 36. B 37. B 38. B 39. D 40. D


25
41. D 42. D 43. D 44. C 45. D
B. 3
4
46. C 47. A 48. C 49. A 50. C
C. 1
25

D. 1
5 Facebook Study Group

50. A basket contains 9 apples, 8 If you need an explanation to


bananas and 7 oranges. A fruit is any of the answers or you want to
picked from the basket, find the download more past questions
probability that it is neither an for FREE.
apple nor an orange.
Click to ENTER our Facebook

A. 7 study group
24

B. 2
3

C. 3
8

D. 1
3

pdfmadeazy.com.ng
2014 JAMB MATHEMATICS QUESTIONS

TYPE: S 4. Express the product of 0.00043


and 2000 in standard form.
1. Which Question Paper Type of
Mathematics is given to you? A. 8.6 x 10
B. 8.3 x 10-3
A. Type F C. 8.6 x 10-2
B. Type E D. 8.6 x 10-1
C. Type L
D. Type S 5. A man donates 10% of his
monthly net earnings to his
2. Find the value of 1101112 + church. If it amounts to ₦4,500,
101002 what is his net monthly income?

A. 10011112 A. ₦62,500
B. 11010112 B. ₦40,500
C. 1001012 C. ₦45,000
D. 10011112 D. ₦52,500

3. A woman bought a grinder for 6. If log 7.5 = 0.8751, evaluate 2


₦60,000. She sold it at a loss of log 75 + log 750.
15%. How much did she sell it?
A. 66.253
A. ₦50,000 B. 6.6252
B. ₦53,000 C. 6.6253
C. ₦52,000 D. 66.252
D. ₦51,000
7. Solve for x in 8x-2 = 2
25

pdfmadeazy.com.ng
A. 10
B. 4
C. 6
D. 8

8. Simplify 2√2−√3
√2+√3

11. From the Venn diagram


A. 3√6 + 1 above, the shaded parts represent
B. 3√6 − 7
C. 3√6 + 7
D. 3√6 − 1 A. (𝑃 ∩ 𝑄) ∩ (𝑃 ∩ 𝑅)
B. (𝑃 ∩ 𝑄) 𝖴 (𝑃 ∩ 𝑅)
9. Evaluate log2 8 + log2 16 − C. (𝑃 𝖴 𝑄) ∩ (𝑃 𝖴 𝑅)
log2 4 D. (𝑃 𝖴 𝑄) 𝖴 (𝑃 𝖴 𝑅)

A. 6 12. If gt2 − k − w = 0, make g


B. 3 the subject of the formula
C. 4
D. 5 A.
k−w
t
k+w
B. t2

10. If P = {1,2,3,4,5} and P 𝖴 Q C.


k−w
t2

= {1,2,3,4,5,6,7}, list the D.


k+w
t

elements in Q
13. Factorize 2y2 - 15xy + 18x2
A. {5, 7}
B. {6} A. (3y + 2x) (y - 6x)
C. {7} B. (2y − 3x) (y + 6x)
D. {6, 7} C. (2y - 3x) (y −6x)

pdfmadeazy.com.ng
D. (2y + 3x) (y − 6x) 17. What is the solution of x−5 <
x+3

−1?
14. Find the value of k if y −1 is a
factor of y3 + 4y2 + ky − 6 A. x < -3 or x > 5
B. -3 < x < 1
A. 0
C. x < -3 or x > 1
B. -6
D. -3 < x < 5
C. -14
D. 1 18. Evaluate the inequality x + 3 ≤
2 4
5x 7
2 −
15. y varies directly as w . When 6 12

y = 8, w = 2. Find y when w = 3
A. x ≥−4

A. 6 B. x ≥ 4

B. 18 C. x ≤ 3

C. 12 D. x ≥ -3

D. 9
19. The 4th term of an A.P. is 13

16. P varies directly as Q and while the 10th term is 31. Find the

inversely as R. When Q = 36 and 24th term.

R = 16, P = 27. Find the relation


between P, Q and R. A. 69
B. 89

A. P=12 C. 75
QR
D. 73
B. P= Q
12R

C. P=12Q 20. What is the common ratio of


R

D. P=12QR the G.P. (√10 + √5)(√10 + 2√5)


+ ⋯?

pdfmadeazy.com.ng
A. 5 0 3 2
24. Find the value of |1 7 8|
B. √2 0 5 4
C. √5
D. 3 A. −2
B. 12
21. A binary operation * is C. 10
defined by x * y = xy. If x * 2 = D. −1
12 − x, find the possible values of
x 25. How many sides has a regular
polygon whose interior angle is
A. −3, −4 135⁰ each?
B. 3, 4
C. 3, −4 A. 8
D. −3, −4 B. 12
C. 10
22. Find y, if 5 −6 x 7 D. 9
( ⅁ )( ) = ( )
2 −7 y −11

A. 2
B. 8
C. 5
D. 3

−x 12
23. If │ │ = −12, find x
−1 4

A. 6 26. In the figure above, KLǁNM,

B. −6 LN bisects <KNM. If angles KLN is

C. −2 54⁰ and angle MKN is 35⁰,


D. 3 calculate the size of angle KMN.

pdfmadeazy.com.ng
A. 19° 29. If the angle of a sector of a
B. 91° circle with radius 10.5 cm is 120°,
C. 89° find the perimeter of the sector.
D. 37°
A. 2.5m
B. 8.0m
C. 7.5m
D. 5.0m

30. A cylindrical tank has a


27. From the figure above, what capacity of 6160m3. What is the
is the value of p? depth of the tank if the radius of
its base is 28cm? [π = 22]
7
A. 135°
B. 90⁰ A. 8.0m
C. 60⁰ B. 7.5m
D. 45⁰ C. 5.0m
D. 2.5m

31. The locus of a dog tethered to


a pole with a rope of 4m is a

28. Find the value of x in the


figure above A. semi-circle with radius 4m
B. circle with diameter 4m

A. 4√3 cm C. circle with radius 4m

B. 120√3 cm D. semi-circle with diameter 4m

C. 10√3 cm
D. 5√3 cm 32. Find the mid-point of S(−5, 4)
and T(−3, −2)

pdfmadeazy.com.ng
A. 4, −1 B. y = x + 5
B. −4, 2 C. y = -x + 5
C. 4, −2 D. y = x – 5
D. −4, 1
35. Calculate the mid-point of the
33. The gradient of a line joining line segment y − 4x + 3 = 0,
(x,4) and (1,2) is 1. Find the value which lies between the x-axis and
2

of x y-axis.

A. −5
2 3
B. 5 A. (− , )
3 2
3 3
C. 3 B. ( , − )
8 2
D. −3 3 3
C. ( , )
8 2
3 3
D. (− , )
2 2

36. Find the equation of the


straight line through (-2, 3) and
perpendicular to 4x + 3y - 5 = 0

A. 5x - 2y - 11 = 0
B. 3x - 4y + 18 = 0
C. 3x + 2y - 18 = 0
34. In the figure above, what is D. 4x + 5y + 3 = 0
the equation of the line that
passes the y-axis at (0,5) and 37. If sin θ = 12, find the value of
13
passes the x-axis at (5,0)?
1 + cos θ

A. y = -x − 5
A. 5
13
pdfmadeazy.com.ng
B. 25 A. -cos 2𝑥 + k
13
C. 18 B. cos 2𝑥 + k
13
C. 1 cos 2𝑥 + k
D. 8 2
13
D. −1 cos 2𝑥 + k
2

38. If y = 4x3 - 2x2 + x, find dy


dx
1
42. Evaluate ∫(2x + 3) dx 2

A. 12x2 − 4x + 1
3
2
B. 8x − 2x + 1 A. 1 (2𝑥 + 3)4 + k
12

C. 8x2 − 4x + 1 B. 1 (2𝑥 + 3)6 + k


12
D. 12x2 −2x + 1 1
C. 1 (2𝑥 + 3)42 + k
3
1 3
39. If y = cos 3x, find dy D. (2𝑥 + 3)2 + k
dx 3

A. −3sin 3𝑥
B. 1 sin 3𝑥
3

C. −1 sin 3𝑥
3

D. 3 sin 3𝑥

40. Find the minimum value of y


= x2 − 2x−3

A. −4
43. The pie chart above shows the
B. 4
monthly distribution of a man's
C. 1
salary on food items. If he spent
D. −1
₦8,000 on rice, how much did he
spend on yam?
41. Evaluate ∫ sin 2𝑥 dx

pdfmadeazy.com.ng
A. ₦42,000 D. 4
B. ₦18,000
C. ₦16,000 47. Find the standard deviation of
D. ₦12,000 5, 4, 3, 2, 1

44. The mean of 2 - 4, 4 + t, 3 - A. √10


2t and t - 1 is B. √2
C. √3
A. −2 D. √6
B. t
C. −t 48. In how many ways can a
D. 2 team of 3 girls be selected from 7
girls?
Values 0 1 2 3 4
Frequency 1 2 2 1 9 7!
A.
2!5!
7!
45. Find the mode of the B. 3!
7!
distribution above C. 4!
7!
D. 3!4!

A. 4
B. 1 Numbers 1 2 3 4 5 6
Frequency 18 22 20 16 10 14
C. 2
D. 3 49. The table above represents
the outcome of throwing a die
46. Find the median of 5, 9, 1, 100 times. What is the probability
10, 3, 8, 9, 2, 4, 5, 5, 5, 7, 3 and of obtaining at least a 4?
6 A. 3
4
A. 3
B. 1

B. 6 5

C. 5

pdfmadeazy.com.ng
C. 1
2

D. 2
5

50. A number is chosen at


random from 10 to 30 both
inclusive. What is the probability
that the number is divisible by 3?

A. 3
5

B. 2
15

C. 1
10

D. 1
3

ANSWERS

1. D 2. D 3. D 4. D 5. D 6. C 7. A
8. B 9. D 10. D 11. A 12. B 13. C
14. D 15. B 16. C 17. B 18. B
19. D 20. B 21. C 22. D 23. A
24. A 25. D

26. B 27. C 28. C 29. C 30. A


31. B 32. D 33. B 34. B 35. D
36. B 37. C 38. A 39. A 40. C
41. D 42. D 43. D 44. D 45. A
46. C 47. B 48. D 49. D 50. D

pdfmadeazy.com.ng
2015 JAMB MATHEMATICS QUESTIONS

1. The sum of the progression is acts on the particle in the


1+x+x2+…= direction north-east. The resultant
of the two forces is

A. 1
1−x
B. 1 A. √3 𝑢𝑛i𝑡𝑠
1+x
B. 3√𝑢𝑛i𝑡𝑠
C. 1
x−1
C. √41 + 20√2 𝑢𝑛i𝑡𝑠
D. 1
x D. √41 + 202 𝑢𝑛i𝑡𝑠

2. Find a square root of 170- 20


1 1
(a−a )(a+
a )
5. Simplify 1
(a2− 2 )
√30 a

2/
A. 2√10 − 5 A. a 3

1/
B. 3√5 − 8√6 B. a− 2

C. 2√5 − 5√6 C. a
1/
5

D. 5√5 −2√6 1/
D. a 3

3. Multiply (x+3y + 5) by (2x2 +


6. In the diagram below PQ is
5y + 2)
parallel to RS, calculate the value
of x
A. 2x +3yx +10xy+15y +13y+10x +2x+10
3 2 2 2

B. 2x3+6yx2+5xy+15y2 +31y+10x2+2x+10

C. 2x3 +3yx2+5xy+10y2+13y+5x2+2x+10

D. 2x3 +6yx2+5xy+15y2+13y+10x2+2x+10

4. A force of 5 units acts on a


particle in the direction to the
east and another force of 4 units

pdfmadeazy.com.ng
A. 20° days, he spends X cedis per day.
B. 40° The amount he has to spend per
C. 60° day for the rest of his stay is
D. 80°
A. Y(y+x) cedis
y−x
7. A ladder resting on a vertical
B. Yy+Xx 𝑐e𝑑i𝑠
wall makes an angle whose y−x

C. Y−xy
𝑐e𝑑i𝑠
tangent is 2.4 with the ground. If x−y

the distance between the foot of D. Y−Xx


y−x
the ladder and the wall is 50cm,
what is the length of ladder?
10. The mean of the numbers 1.2,
1.0, 0.4, 1.4, 0.8, 0.8, 1.2 and
A. 1.3m
1.1 is
B. 1.1m
C. 1.2m
A. 1.5
D. 1.3m
B. 0.8
C. 1.0
8. After getting a rise of 15%, a
D. 1.05
man’s new monthly salary is
₦345. How much per month did
11. A solid cylinder of radius 3cm
he earn before the increase?
has a total surface area of 36cm2.
Find its height
A. ₦350
B. ₦396.75
A. 2cm
C. ₦300
B. 3cm
D. ₦293.25
C. 4cm
D. 5cm
9. A trader goes to Ghana for y
days with Y cedis. For the first x

pdfmadeazy.com.ng
12. When a dealer sells a bicycle 15. A pentagon has four of its
for ₦81, he makes a profit of 8%. angles. If the size of the fifth
What did he pay for the bicycle? angle is 60°. Find the size of each
of the four equal angle is
A. ₦75
B. ₦74.52 A. 60°
C. ₦75 B. 108°
D. ₦75.52 C. 120°
D. 150°
13. Which of the formula below
represents the general terms of 16. In the figure below PQ//SR,
the following set of numbers? ST//RQ, PS = 7cm, PT=7cm, SR=
{−1, 2 ,− 1 , 2
,…} for n = 1, 2, 3, 4cm. Find the ratio of the area of
3 2 5

4, … QRST to the area of PQRS

A. 2
n−1

B. (-)n+1 2
n+1
2
C. (−)𝑛
n+1

D. n
2n−1

A. 56:77
14. Write the decimal number 39 B. 56:105
to base 2 C. 28:105
D. 28:49
A. 100111
B. 110111 17. Find a two-digit number such
C. 111001 that three times the ten digits is 2
D. 100101 less than twice the units digit and
twice the units digit and twice the

pdfmadeazy.com.ng
number is 20 greater than the 5, 3, 6, 9, 4, 7, 8, 6, 2, 7, 8, 4, 5,
number obtained by reversing the 2, 1, 0, 6, 9, 0, 8. The arithmetic
digits. mean of the marks is

A. 24 A. 6
B. 42 B. 5
C. 74 C. 7
D. 47 D. 4

18. In the figure above, the area 21. The graphical methods of
of XYZW is solving the equation x3 + 3x2 +4x
−28 =0 is by drawing the graphs
A. 60 cm2 of the curves
B. 54 cm2
C. 27 cm2 A. y=x3 and y= 3x2 + 4x −48
D. 52.2 cm2 B. y= x3 +3x2+ 4x−28 and the
line y =1
19. In ΔXYZ, XY = 3 cm, XZ= 5 C. y=x3 + 3x2 +4x and y = 28 𝑥
cm and YZ= 7cm. If the bisector D. y= x2 + 3x + 4 and y = 28 𝑥
of XYZ meets XZ at W, what is
the length of XW? 22. A sector of a circle is bounded
by two radii 7 cm long and an arc
A. 1.5 cm of length 6cm. Find the area of
B. 2.5 cm the sector.
C. 3 cm
D. 4cm A. 42 cm2
B. 3 cm2
20. Marks scored by some C. 21 cm2
children in an arithmetic test are D. 24 cm2

pdfmadeazy.com.ng
23. Express 150 kilometres per Representing the above on a Pie
second in metres per hour. Chart, the angle of the sector
representing the participants from
A. 7.8 × 105 Asia is
B. 4.5 × 106
C. 7,800,000 A. 150°
D. 4.68 × 106 B. 67.5°
C. 67°
24. The arithmetic mean of the D. 135°
ages of 30 pupils in a class is 15.3
years. One boy leaves the class 26. Find the sum to infinity of the
and one girl is enrolled, and the following sequence: 1, 9 10 ,
new average age of 30 pupils in 9 102, 9 103
the class becomes 15.2 years.
How much older is the boy than A. 1 10
the girl? B. 9 10
C. 10 9
A. 30 years D. 10
B. 6 years
C. 9 years 27. Which of the following is a
D. 3 years sketch of y =3 sin x?

25. A world congress of


mathematician was held in Nice in
1970 with 800 people
participating. There were 300
from Europe, 200 from America,
150 from Asia, 45 from Africa and
105 from Australia.

pdfmadeazy.com.ng
28. Given that log𝑎 2 = 0.693 and calculate the area of the sector
log𝑎 3 =1/097, Find log𝑎 13.5 subtended by the given angle.

A. 1.404 A. 22 cm2
B. 1.790 B. 44 cm2
C. 2.598 C. 66 cm2
D. 2.790 D. 88 cm2

29. If the function f (𝑥) = 𝑥3 + 2𝑥2 32. The angle of elevation of a


+ 𝑞𝑥 − 6 is divisible by the factor building from a measuring
x+1, find q. instrument placed on the ground
is 30°. If the building is 40m high,
A. −5 how far is the instrument from
B. −2 the foot of the building?
C. 2
D. 5 A. 20 m
√3

B. 40 m
√3
30. What value of g will make the
C. 20√3 m
expression 4x2 −18xy + y +g a
D. 40√3 𝑚
perfect square?

33. Integrate 1
+ cos 𝑥 with
A. 9 x

B. 9y2 respect to x
4
C. 81𝑦2
A. − 1 𝑥 + sin 𝑥 + 𝑘
81y2
D.
4 B. 𝐼𝑛 𝑥 + sin 𝑥 + 𝑘
C. In x -sin 𝑥 + k
31. An arc of a circle subtends an D. −1 8 sin x + k
angle 70° at the centre. If the
radius of the circle is 6cm,
pdfmadeazy.com.ng
34. dy
dx
cos (3𝑥2 − 2𝑥) is equal to B. (−2 −1)
−1 −1
−1 −1
C. ( )
0 −1
A. −sin(6𝑥 − 2) −1 1
2 D.
B. −sin(3𝑥 − 2) ( )
0 1
C. (6x−2) sin(3𝑥2 − 2𝑥)
D. −(6x−2) sin(3𝑥2 − 2𝑥) 38. The first term of a geometrical
progression is twice its common
35. If log8 10 = x, evaluate log8 5 ratio. Find the sum of the first two
in terms of x. terms of the progression if its sum
to infinity is 8.
A. 1 2x
B. 𝑥 − 1 4 A. 8 5
C. x −1 3 B. 8 3
D. x −1 2 C. 72 25
D. 56 9
0.0023 × 750
36. Simplify √
0.00345 × 1.25
39. In ΔMNO, MN = 6 units, MO
= 4 units and NO =12 units. If
A. 15
the bisector of angle M meets NO
B. 20
at P, calculate NP.
C. 40
D. 75
A. 4.8 units
B. 7.2 units
37. Find the matrix T if ST = I
C. 8.0 units
−1 1
where S = ( ) and I is the D. 18.0 units
1 −2
identity matrix
40. Evaluate ∫ n (sin 𝑥 − cos 𝑥) 𝑑𝑥
4
−2 1
A. ( )
−1 1
A. √2 + 1

pdfmadeazy.com.ng
B. √2 − 1
C. −√2 − 1
D. −√2

ANSWERS

1. A 2. C 3. B 4. C 5. D 6. B 7. D
8. C 9. D 10. C 11. C 12. B 13. C
14. A 15. C 16. B 17. D 18. C
19. A 20. B

21. D 22. C 23. D 24. D 25. B


26. D 27. D 28. C 29. A 30. D
31. A 32. D 33. B 34. D 35. C
36. B 37. C 38. B 39. B 40. B

Facebook Study Group

If you need an explanation to


any of the answers or you want to
download more past questions
for FREE.

Click to ENTER our Facebook


study group

pdfmadeazy.com.ng
2016 JAMB MATHEMATICS QUESTIONS

1. Without using tables, evaluate


log2 4 + log4 2 − log25 5

A. 1 2
B. 1 5
C. 0 4. In ΔXYZ above, X⏞𝐾𝑍 = 90°,
D. 2 X𝐾 = 15 𝑐𝑚, X𝑍 = 25𝑐𝑚 𝑎𝑛𝑑 𝑌𝐾 =
8 𝑐𝑚. Find the area of ΔXYZ.
2. Find the values of p for which
the equation 𝑥2−(p−2) x + 2p+ A. 180 sq.cm
1= 0 has equal roots. B. 20sq cm
C. 160sq. cm
A. (0, 12) D. 320 sq.cm
B. (1, 2)
C. (21, 0) 5. Simplify 31 − 11 × 2 + 12
3 4 3 5
D. (4, 5)

A. 217
3. Solve the simultaneous 30

B. 3 9
equation 2𝑥 − 3𝑦 =10, 10𝑥 − 6𝑦 10

=5 C. 4 1
10

D. 411
1 1 36
A. 𝑥 = 2 , 𝑦 = 3
2 3

B. 𝑥 = 31, 𝑦 = 21 6. Factorize 1 − (𝑎 − 𝑏)2


2 2

C. 𝑥 = 21, 𝑦 = 3
4
A. (1 − 𝑎 − 𝑏)(1 − 𝑎 + 𝑏)
D. 𝑥 = 31, 𝑦 = 21
2 5 B. (1 − 𝑎 + 𝑏)(1 + 𝑎 − 𝑏)
C. (1 − 𝑎 + 𝑏)(1 − 𝑎 + 𝑏)
D. (1 − 𝑎 − 𝑏)(1 + 𝑎 − 𝑏)

pdfmadeazy.com.ng
7. Find the range of value of x C. 𝑦 = 𝑥2 + 7𝑥 + 9
which satisfy the inequality D. 𝑦 = 𝑥2 + 14𝑥 + 11
x x x
( + + )<1
2 3 4
10. Differentiate (cos 𝑞 − sin 𝑞)2
with respect to q.
A. 𝑥 < 12
13

B. 𝑥 < 13
A. −2 cos 2𝑞
C. 𝑥 < 3
B. −2 sin 2𝑞
D. 𝑥 < 13
12 C. 1 − 2 cos 2𝑞
D. 1 − 2 sin 2𝑞
8. A crate of soft drinks contains
10 bottles of Cococola, 8 of Fanta 11. If tan 𝑞 = 5 4 , find 𝑠i𝑛2𝑞 −
and 6 of Sprite, if one bottle is 𝑐o𝑠2𝑞.
selected at random, what is the
probability that it is NOT a Coca- A. 5 4
Cola bottle? B. 41 9
C. 9 41
A. 5 12 D. 1
B. 1 3
C. 3 4 12. Find the value of x if the
D. 7 12 expression 𝑘𝑥2 + 𝑥 − 5𝑥 − 2
leaves a remainder 2 when it is
9. The gradient of curve is 2𝑥 + 1 divided by 2𝑥 + 1.
and the curve passes through
point ( 2, 0). Find the equation of A. 10
the curve. B. 8
C. −10
A. 𝑦 = 𝑥 + 7𝑥 + 9
2
D. −8
B. 𝑦 = 𝑥 + 7𝑥 − 18
2

pdfmadeazy.com.ng
13. If 𝑦 = 𝑥2 − 𝑥 − 12, find the 16. The sum of the first n terms
range of values of x for which y ≥ of the arithmetic progression 5,
0. 11, 17, 23, 29, 35,… is

A. 𝑥 < −2 o𝑟 𝑥 > 4 A. 𝑛(3𝑛 − 0)


B. 𝑥 ≤ −3 o𝑟 𝑥 ≥ 4 B. 𝑛(3𝑛 + 2)
C. −3 < 𝑥 ≤ 4 C. 𝑛(3𝑛 + 2.5)
D. −3 ≤ 𝑥 ≤ 4 D. 𝑛(3𝑛 + 5)

14. A man bought a second-hand 17. What value of 𝑥 will make the
photocopying machine for ₦34 function 𝑥(4 − 𝑥) a maximum?
000. He served it at a cost of
₦2,000 and then sold it at a profit A. 4
of 15%. What was the selling B. 3
price? C. 2
D. 1
A. ₦37, 550
B. ₦40, 400 18. In how many ways can a
C. ₦41, 400 delegation of 3 be chosen from 5
D. ₦42, 400 men and 3 women, if at least 1
man and 1 woman must be
15. Find the radius of a sphere included?
whose surface area is 154 𝑐𝑚2
A. 15
A. 7.00 cm B. 28
B. 3.50 cm C. 30
C. 3.00 cm D. 45
D. 1.75cm

pdfmadeazy.com.ng
19. The table above shows the D. 3√15 + 11
distribution of marks of students
in attest. Find the probability of 22. Solve the inequalities −6 ≤ 4
passing the test if the pass mark — 2𝑥 < 5 − 𝑥
is 5
A. −1 ≤ 𝑥 < 6
A. 3/5 B. −1 < 𝑥 ≤ 5
B. 4/9 C. −1 < 𝑥 < 5
C. 7/20 D. −1 ≤ 𝑥 ≤ 6
D. −1/5
23. A cylindrical pipe 5cm long
20. A student measures a piece of with radius 7m has one end open.
rope and found that it was 1.26m What is the total surface area of
long. If the actual length of the the pipe?
rope was 1.24 m, what was the
percentage error in the A. 100𝜋𝑚2
measurement? B. 98𝜋𝑚2
C. 350𝜋𝑚2
A. 0.40% D. 749𝜋𝑚2
B. 0.01%
C. 0.25% 24. Find the standard deviation of
D. 0.80% 2, 3, 5 and 6

21. Rationalize 2√3+√5 A. √5 2


√5−√3
B. √10

3√12+11 C. √6
A.
2
D. √2 5
3√15−11
B.
2

C. 3√15 − 11

pdfmadeazy.com.ng
25. Without using tables evaluate 28. Divide 2𝑥3 + 11𝑥2 + 17𝑥 + 6
(343)−1 3 × (0.14) − 1 × (25)−1 2 𝑏𝑦 2𝑥 + 1

A. 10 A. 𝑥2 + 5𝑥 + 6
B. 12 B. 2𝑥2 + 5𝑥 + 6
C. 8 C. 2𝑥2 − 5𝑥 + 6
D. 7 D. 𝑥2 − 5𝑥 + 6

26. Given that log4 (𝑦 − 1) + log4 ⨂ p q r s


p r p r p
(1 2 𝑥) =1 𝑎𝑛𝑑 log2 (𝑦 + 1) + log2
q p q r s
𝑥=2, solve for x and y r r r r r
s q s r q
respectively.

29. The identity element with


A. 2, 3
respect to the element shown in
B. 3, 2
the table above is
C. −2, −3
D. −3, −2
A. p
B. q
27. When the expression 𝑝𝑚2 +
C. r
𝑞𝑚 + 1 is divided by (m-1), it has D. s
a remainder 2 and when divided
by (m+1) the remainder is 4. Find
p and q respectively.

A. 2, -1
B. -1, 2
C. 3, -2 30. In the figure above, PQST is a

D. -2, 3 parallelogram and TSR is a


straight line. If the area of ΔQRS

pdfmadeazy.com.ng
is 20𝑐𝑚2, find the area of the 33.
0.0001432
= 𝑘 x 10𝑛 wℎe𝑟e 1 £ 𝐾
1940000
trapezium PQRT < 10 𝑎𝑛𝑑 𝑛 i𝑠 𝑎 wℎo𝑙e 𝑛𝑢𝑚𝑏e𝑟. The
value of 𝐾 𝑎𝑛𝑑 𝑛 are
A. 35𝑐𝑚2
B. 65𝑐𝑚2 A. 7.381 and -11
C. 70𝑐𝑚2 B. 2.34 and 10
D. 140𝑐𝑚2 C. 3.871 and 2
D. 7.831 and -11
31. The mid-point of the segment
of the line 𝑦 = 12 × 15 which lies 34. Thirty boys and x girls sat for
between the x-axis and y-axis is a test. The mean of the boys’
A. (−2 , 3) scores and that of the girls were
2 2

B. (−2 , 3) respectively 6 and 8. Find 𝑥 if the


3 2

C. (3, 3) total score was 468.


8 2
D. (−3, 3)
6 2 A. 38
B. 24
32. Find the equation of the curve C. 36
which passes through the point D. 22
(2, 5) and whose gradient at any
point is given by 6𝑥 − 5 35. Rationalize 5√7−7√5
√7− √5

A. 6𝑥2 − 5𝑥
A. −2√35
B. 6𝑥2 + 5𝑥 + 5
B. √7 − 6√5
C. 3𝑥2 − 5𝑥 − 5
C. −√35
D. 3𝑥2 − 5𝑥 + 3
D. 4√7

36. If 2𝑥 + 3𝑦 = 1 𝑎𝑚𝑑 𝑥 − 2𝑦 =
11, fi𝑛𝑑(𝑥 + 𝑦)
pdfmadeazy.com.ng
A. 5 40. Find the number of sides of a
B. -3 regular polygon whose interior
C. 8 angle is twice the exterior angle.
D. 2
A. 2
37. A car dealer bought a second- B. 3
hand car for ₦250,000.00 and C. 6
spent ₦70,000.00 refurbishing it. D. 8
He then sold the car for
₦400,000.00. What is the ANSWERS
percentage gain?
1. A 2. A 3. A 4. B 5. B 6. B 7. A
3
38. Simplify ( √6403 )
−1
8. D 9. C 10. A 11. A 12. C 13. B
14. C 15. B 16. B 17. C 18. D

A. 80 19. A 20. A

B. 40
21. A 22. B 23. D 24. A 25. A
C. 1 40
D. 1 80 26. C 27. A 28. A 29. C 30. C
31. B 32. D 33. A 34. C 35. C
39. Find the value of p if the line
joining (p, 4) and (6, -2) is 36. D 37. B 38. C 39. B 40. C

perpendicular to the line joining


(2, p) and (-1, 3)

A. 0
B. 3
C. 4
D. 6

pdfmadeazy.com.ng
2017 JAMB MATHEMATICS QUESTIONS

1. If dy
= 2𝑥 − 3 𝑎𝑛𝑑 𝑦 = 3 wℎe𝑛 𝑥 4. Calculate the mean deviation of
dx
the sets of numbers 7, 3, 14,9,7
= 0, 𝐹i𝑛𝑑 𝑦 i𝑛 𝑡e𝑟𝑚𝑠 of 𝑥
and 8.

A. 2𝑥2 − 3𝑥
A. 21
B. 𝑥2 — 3𝑥 6

C. 𝑥2 − 3𝑥 − 3 B. 21
2
2
D. 𝑥 − 3𝑥 + 3 C. 11
6

2 1 D. 21
2. If P= ( ) and I is a 2×2- 3
−3 0
unit matrix. Evaluate 𝑃2 — 2𝑃 +
5. The graph of the function 𝑦 =
41
𝑥2 + 4 and a straight-line PQ are
drawn to solve the equation 𝑥2 −
9
4
A. ( )
3𝑥 + 2 = 0. What is the equation
−12 1
PQ?
B. (−3 0 )
0 −3
1 0
C. ( )
0 1 A. 𝑦 = 3𝑥 − 2
D. (1 4) B. 𝑦 = 3𝑥 + 2
4 1
C. 𝑦 = 3𝑥 − 4
D. 𝑦 = 3𝑥 + 4
3. Find the value of x if the line 2𝑦
— 𝑎𝑥 + 4 = 0 is perpendicular to
6. If ≤ 𝜃 < 2𝜋, fi𝑛𝑑 𝑡ℎe 𝑚𝑎𝑥i𝑚𝑢𝑚
the line 𝑦 + 1 𝑥 — 1 = 0 2
4 4
𝑣𝑎𝑙𝑢e of f(𝜃) =
6+2005𝜃

A. −4
B. 4 A. 4

C. 8 B. 1

D. −8 C. 2 3
D. 1 3
pdfmadeazy.com.ng
7. On a pie chart, there are four B. {2, 10, 14, 22, 26}
scores of which three angles are C. {2, 4, 14, 18, 26}
45°, 90° and 35°, if the smallest D. {0, 2, 6, 22, 26}
sector represents ₦28.00, how
much is the largest sector? 10. 4 2 4 3
-1 3 𝑥 4
A. ₦96.00 -------------------
B. ₦84.00 Y 3 4 4
C. ₦48.00 --------------------
D. ₦42.00
Find x and y respectively in the
8. The result of missing a fair dice subtraction above carried out in
120 times is summarize is above, base 5.
find the value of x
A. 2, 4
A. 19 B. 3, 2
B. 20 C. 4, 2
C. 21 D. 4, 3
D. 22
11. Factorise completely x-2(𝑥 −
9. Given μ = {e𝑣e𝑛 𝑛𝑢𝑚𝑏e𝑟 𝑎2 + 4𝑏2)
𝑏e𝑡wee𝑛 0 𝑎𝑛𝑑 30}
P={𝑀𝑢𝑙𝑡i𝑝𝑙e𝑠 of 6 𝑏e𝑡wee𝑛 0 𝑎𝑛𝑑 A. (𝑎 − 2𝑏)(𝑐 + 𝑎 − 2𝑏)
30} B. (𝑎 − 2𝑏)(−𝑎 − 2𝑏)
Q={𝑀𝑢𝑙𝑡i𝑝𝑙e𝑠 of 4 𝑏e𝑡wee𝑛 0 𝑎𝑛𝑑 C. (𝑎 − 2𝑏)(𝑐 + 𝑎 − 2𝑏)
30} D. (𝑎 − 2𝑏)(𝑐 − 𝑎 + 2𝑏)
Find (P𝖴Q)
12. PQRSTN is a regular polygon
A. {2, 10, 14, 22, 26} of side 7cm. Inscribed in a circle,

pdfmadeazy.com.ng
find the circumference of the tonnes of millet is harvested,
circle PQRSTV what amount of beans is
harvested?
A. 12cm
B. 42cm A. 9000 tonnes
C. 44cm B. 6000 tonnes
D. 56cm C. 1500 tonnes
D. 1200 tonnes
13. The shadow of a pole 5√3
high is 5.5cm. Find the angle of 15. A container has 30 gold
elevation of the sum. medals, 22 silver medals and 18
bronze medals. If one medal is
A. 50 selected at random from the
B. 45 container, what is the probability
C. 60 that it is not a gold medal?
D. 75
A. 4/7
B. 3/7
C. 11/35
D. 9/35

16. A polynomial in x whose zeros


are −2, −1 𝑎𝑛𝑑 3 i𝑠

A. 𝑥3 − 7𝑥 + 6
B. 𝑥3 + 7𝑥 − 6
14. The pie chart above shows the C. 𝑥3 + 7𝑥 + 6
distribution of the crops harvested D. 𝑥3 − 7𝑥 − 6
from a farmland in a year. If 3000

pdfmadeazy.com.ng
17. If M and N are two matrices A. 226°
1 3 2 B. 224°
defined by M= ( 4 5 −1) and
−3 2 0 C. 136°
1 −2 3 D. 134°
N= (4 −1 5 ). Evaluate 2M-3N
2 −3 −1
19. The mean height and range of
−1 12 5 heights 1.35, 1.25, 1.35, 1.40,
A. ( 4 7 3)
1.35, 1.45, 1.50, 1.35, 1.50 and
0 −5 −3
−1 0 −5 1.20 are m and r respectively.
B. (−4 7 −17) Find 𝑚 + 2𝑟.
0 −5 3
−1 12 −5
C. ( −4 −13 −17) A. 1.35
−12 13 3
−1 12 −5 B. 1.65
D. ( 4 13 13) C. 1.95
−12 13 3
D. 3.00

20. What is the probability that an


integer x, {1 ≤ 𝑥 ≤ 20} chosen at
random is divisible by both 2 and
3?

A. 1/20
B. 1/3
C. 3/20
D. 7/20

18. From the diagram above, find 21. A trader bought goats for ₦
the bearing of R from S. 4000 each. He sold them for

pdfmadeazy.com.ng
₦180,000 at a loss of 25%. How A. 36°
many goats did he buy? B. 18°
C. 72°
A. 60 D. 24°
B. 50
C. 45 24. If tan θ = 4 3, calculate 𝑠i𝑛2𝜃
D. 36 — 𝑐o𝑠2𝜃

22. Evaluate
0.21 × 0.072 × 0.00054
A. 16 25
0.006 × 1.68 × 0.063
B. 24 25

A. 0.01286 C. 7 25

B. 0.01285 D. 9 25

C. 0.1286
3 5 −4
D. 0.1285 25. If N= [ 6 −3 −5] find |𝑁|
−2 2 1

A. 17
B. 23
C. 65
D. 91

26. A man invested ₦5, 000 for 9


months at 4%. What is the simple
interest?

23. In the diagram above, PST is A. ₦150

a straight line, PQ = QS = RS. If B. ₦220

<RST = 72°, find 𝑥 C. ₦130


D. ₦250

pdfmadeazy.com.ng
27. Rationalize 2−√5 A. 56 cm2
3−√5
B. 49 cm2

1−√5
C. 98 cm2
A.
2 D. 28 cm2
1−√5
B.
4
√5−1
31. If two graphs 𝑦 = 𝑝𝑥2 + 𝑞 𝑎𝑛𝑑
C.
2 𝑦 = 2𝑥2 −1 i𝑛𝑡e𝑟𝑠e𝑐𝑡 𝑎𝑡 𝑥 = 2. Find
1+√5
D.
4 the value of 𝑝 in terms of 𝑞?

28. Factorize completely 9𝑦2−16𝑥2 A.


7+q
8
8−q
B. a
q−8
A. (3𝑦 − 2𝑥)(3𝑦 + 4𝑥) C. 7
7−q
B. (3𝑦 + 4𝑥)(3𝑦 + 4𝑥) D. 4

C. (3𝑦 + 2𝑥)(3𝑦 − 4𝑥)


D. (3𝑦 + 4𝑥)(3𝑦 − 4𝑥) 32. A straight line makes an angle
of 30° with the positive 𝑥 − 𝑎𝑥i𝑠

29. Solve the inequalities 𝑥2 + 2𝑥 and the 𝑦 − 𝑎𝑥i𝑠 at 𝑦 = 5. Find the

> 15. equation of the straight line.

A. 𝑥 < −3 o𝑟 𝑥 > 5 A. √3 𝑦 = 𝑥 + 5√3

B. −5 < 𝑥 < 3 B. √3𝑦 = −𝑥5√3

C. 𝑥 < 3 o𝑟 𝑥 > 5 C. 𝑦 = 𝑥 + 5

D. 𝑥 > 3 o𝑟 𝑥 < −5 D. 𝑦 = 1 𝑥 +5
10

30. A circle of perimeter 28cm is 33. Find the area bounded by the
opened to form a square. What is curves 𝑦 = 4 − 𝑥2 𝑎𝑛𝑑 𝑦 = 2𝑥 + 1.
the maximum possible area of the
square? A. 101 sq. units
3

pdfmadeazy.com.ng
B. 102 sq. units A. 66°
3

C. 201 sq. units B. 110°


3
C. 26°
D. 202 sq. units
3 D. 70°

34. Teams P and Q are involved


in a game of football. What is the
probability that game ends in a
draw?

A. 1/4
B. 1/3 37. The bar chart above shows

C. 1/2 the mark distribution in a class

D. 2/3 test. Find the number of students


in the class

35. Find without using a logarithm


log3 27−log1 64 A. 9
table the value of 1
4
log
381 B. 2
A. 7 4 C. 60
B. − 7 4 D. 30
C. −3 2
D. 7 3

36. In the figure below, determine


the angle marked 𝑦.
38. What is the volume of the
regular three dimensional figure
drawn above?

A. 160 cm3

pdfmadeazy.com.ng
B. 48 cm3 14. D 15. A 16. D 17. C 18. B
C. 96 cm3 19. C 20. C
D. 120 cm3
21. A 22. A 23. A 24. C 25. A
39. Find the remainder when 𝑥3 − 26. A 27. B 28. D 29. B 30. B
2𝑥2 + 3𝑥 − 3 is divided by 𝑥2 + 1. 31. D 32. A 33. B 34. C 35. C
36. D 37. D 38. B 39. A 40. A
A. 2𝑥 − 1
B. 𝑥 + 3 Facebook Study Group
C. 2𝑥 + 1
D. 𝑥 − 3 If you need an explanation to
any of the answers or you want to
40. In a class of 60 students, 30 download more past questions
offer physics and 40 offer for FREE.
Chemistry. If a student is picked
at random from the class, what is Click to ENTER our Facebook
the probability that the student study group
offers both physics and
Chemistry?

A. 1/3
B. 1/4
C. 1/2
D. 1/6

ANSWERS

1. A 2. C 3. C 4. B 5. C 6. B 7. B
8. B 9. A 10. C 11. C 12. B 13. C

pdfmadeazy.com.ng
2018 JAMB MATHEMATICS QUESTIONS

1. 4 2 4 3 4. If 𝑔𝑡2 − 4 − w = 0 make g the


−1 3 𝑥 4 subject of the formula

u−w
Y 3 4 4 A. t
u+w
B. t2
u−w
Find the 𝑥 𝑎𝑛𝑑 𝑦 respectively in C. t2
u+w
D.
the subtraction above t

5. Find the value of 𝑢 if 𝑦 − 1 i𝑠 𝑎


A. 2, 4
f𝑎𝑐𝑡o𝑟 of 𝑦2 + 4𝑦2 + 4𝑦 − 6
B. 3, 2
C. 4, 2
0
D. 4, 3 A.
-6
B.
-4
2. Express the product of 0.00043 C.
1
in Standard form D.

x 7
6. Find 𝑦 if (5 −6) ( ) = ( )
A. 8.6 × 10
B. 8.6 × 10−3
2 −7 y −1
C. 8.6 × 10 −2

D. 8.6 × 10−1
A. 2
B. 8
3. Simplify 2√5−√3
√2+√3 C. 5
D. 3
A. 5√6 + 1
B. 3√6 − 7 7. In the figure below KL//NM,
C. 3√6 + 7 LN bisects <KNM if the angle KLN
D. 3√6 – 1 is 54° and angle MLN is 35°.
Calculate the size of angle KMN

pdfmadeazy.com.ng
9. In the figure above, what is the
equation of the line that passes
the 𝑦 − 𝑎𝑥i𝑧 𝑎𝑡(0, 5) and passes
the 𝑥 − 𝑎𝑥i𝑠 𝑎𝑡(5, 0)

A. 𝑦 = −𝑥 − 5

A. 108° B. −𝑦 = 𝑥 + 5

B. 91° C. 𝑦 = −𝑥 + 5

C. 84° D. 𝑦 = 𝑥 − 5

D. 37°
10. A construction company is

8. If the angle of a sector of a owned by two partners 𝑥 𝑎𝑛𝑑 𝑦

circle with radius of 10.5cm is and it is agreed that their profit

100. Find the perimeter of the will be divided in the ratio 4:5 at

sector. the end of the 𝑦e𝑎𝑟 𝑦 received


₦5,000 more than x. What is the

A. 2.5 m total profit of the company per

B. 3.0 m year?

C. 7.5 m
D. 5.0 m A. ₦20,000
B. ₦25,000
C. ₦50,000
D. ₦45,000

11. If 𝑥 = 1 i𝑠 𝑡ℎe 𝑟oo𝑡 of 𝑡ℎe


e𝑞𝑢𝑎𝑡io𝑛 𝑥3 − 2𝑥2 − 5𝑥+6. Find
the other roots

A. −3 and 2

pdfmadeazy.com.ng
B. −2 and 2 14. PQRS is a desk of dimension
C. 3 and −2 2m × 0.8m which is inclined at
D. 1 and 3 30° is the horizontal

12. If 𝑥 + 2 𝑎𝑛𝑑 𝑥 − 1 are factors A. 25°, 35'


of the expression 1𝑥3 + 2𝑘𝑥2 +24. B. 30°
Find the value of 𝑙 𝑎𝑛𝑑 𝑘 C. 15°, 36'
D. 10°
A. 𝑙 = −6, 𝑘 = −2
B. 𝑙 = −2, 𝑘 = 1 15. Find the missing point/value
C. 𝑙 = −2, 𝑘 = −1 in the following road
D. 𝑙 = 0, 𝑘 − 1
X -2 -1 0 1 2 3
x2 −3x
Y=x3-x+3 3 3 3 9 27
13. Simplify x−7
×
x2−9 x2−49

A. −5
A. x B. 3
(x−3)(x+7)
(x+3)(x+7) C. −9
B.
x
D. 13
C. x
(x−3)(x−7)

D. x
16. Correct each of the following
(x+3)(x+7)
59.81798 and 0.0746829 to three
significant figures and multiply
them giving your answer to three
significant figures.

A. 4.46
B. 4.48
C. 4.47
D. 4.49

pdfmadeazy.com.ng
17. One interior angle of a convex through the 45km/hr. Find the
of hexagon is 170° and each of average speed for the entire
the remaining angles is equal to journey
𝑥°. Find 𝑥.
A. 55 𝑘𝑚/ℎ𝑟
A. 120° B. 50 𝑘𝑚/ℎ𝑟
B. 110° C. 67.50𝑘𝑚/ℎ𝑟
C. 105° D. 75 𝑘𝑚/ℎ𝑟
D. 102°
21. By selling 20 oranges for
18. If f(𝑥) = 1 + x−1 . Find ₦1.35 a trader makes a profit of
x−1 x2−1

f(1−𝑥) ₦82. Which is his percentage gain


or loss if he sells the same 20

A. 1 +
1 oranges for ₦1.10?
x x−2

B. 𝑥 + 1
2x−1 A. 8%
C.
−1
+
1
B. 10%
x x−2

D.
−1
+
1
C. 12%
x x−1
D. 15%

19. Convert 2415 to base 8


22. Simplify without using tables
2√14 × 3√21
A. 718 7√24 × 2√98
B. 1078
C. 1768 A.
3√14
4
D. 2418 3√2
B.
4
3√14
20. A train moves from P at Q at C.
28

an average speed of 40km/hr and D.


3√2
28
immediately returns from Q is P

pdfmadeazy.com.ng
23. Make y the subject of the A. 213
formula Z=𝑥2 + 1 ? B. 212
y3
C. 211
1 D. 216
A. 𝑦 = (Z−x2)3

B. 𝑦 = 1
26. The histogram above
(Z+x2)3

C. 𝑦 = 1
1
represents the weight of students
(Z−x2)3
who travelled out of their schools
D. 𝑦 = 1
3√Z−3√x2 for an examination. How many
people made the trip?

A. 78
B. 58
C. 29
D. 69

24. A cubic function f (𝑥) is 27. In the many ways can 6

specified by the graph shown subjects be selected from 10

above. The values of the subject be selected from 10

independent variable for which subjects for an examination?

the function x…
A. 218

A.-1, 0, 1 B. 216

B. −1 ≤ 𝑥 ≤ 1 C. 215

C. 𝑥 < −1 D. 210

D. 𝑥 > 1
28. Find the value of x which the
functions f(𝑥) = 2𝑥2 − 𝑥2 − 4𝑥 + 4
25. Find the eleventh term of the
progression 4, 8, 16, … has a maximum value

pdfmadeazy.com.ng
A. 2 3 B. 𝑥 ≤ 3, 𝑥 ≥ 2
B. 1 C. 𝑥 ≤ −2, 𝑥 ≥ −3
C. −2 3 D. 𝑥 ≤ −3, 𝑥 ≥ 2
D. −1

29. Make L the subject of the


42w
formula if 𝑑 = √
5L

42w
A. √
5d

42w
B. √
5d2

42
C. √ 32. In the diagram above PQ//Rs,
5d2

1 42w
The size of the angle marked x is
D. √
2 5

A. 100°

30. Calculate the simple interest B. 80°

on ₦1,500 for 8 years at 5% per C. 50°

annum D. 30°

A. ₦5,000 33. Find the gradient of a line

B. ₦600 which is perpendicular to the line

C. ₦500 with equation 3𝑥 + 2𝑦 + 1 = 1

D. ₦150
A. 3 2

31. Solve the quadratic inequality B. −2 3

𝑥2 − 5𝑥 + 6 ≥ 0 C. −2 5
D. −3 2

A. 𝑥 ≤ 2, 𝑥 ≤ 7

pdfmadeazy.com.ng
34. Evaluate ∫2 cos dx B. 121
−2
C. 181
D. 180
A. 0
B. 1
38. A baking recipe calls for 2.5kg
C. 2
of sugar and 4.5kg of flour. With
D. 3
this recipe some cakes were
baked using 24.5kg of a mixture
35. If the lines 3𝑦 = 4𝑥 − 1 and
of sugar and flour. How much
𝑞𝑦 = 𝑥 + 3 are parallel to each
sugar were used?
other, the value of 𝑞 is

A. 12.25kg
A. −4 3
B. 6.75kg
B. −5 4
C. 8.75kg
C. 4 5
D. 15.75kg
D. 3 4

39. Two cars 𝑥 𝑎𝑛𝑑 𝑦 start at the


36. The volume of a
same point and travel towards a
2
hemispherical bowl is 718 3 𝑐𝑚3, point P which is 150𝑘𝑚 away. If
find its radius the average speed of y is 60𝑘𝑚
per hour and x arrives 𝑎𝑡 𝑃 25
A. 4.0 cm 𝑚i𝑛𝑢𝑡e𝑠 earlier than 𝑦. what is the
B. 5.6cm average speed of 𝑥?
C. 7.0cm
D. 3.6cm A. 513 𝑘𝑚 𝑝e𝑟 ℎo𝑢𝑟
9

B. 72 𝑘𝑚 𝑝e𝑟 ℎo𝑢𝑟
37. If 𝑥 = 1245 , fi𝑛𝑑 𝑥.
C. 66 𝑘𝑚 𝑝e𝑟 ℎo𝑢𝑟
D. 371 𝑘𝑚 𝑝e𝑟 ℎo𝑢𝑟
A. 124 2

pdfmadeazy.com.ng
40. The first term of an Arithmetic
Progression is 3 and the fifth term
is 𝑞. Find the number of terms in
the progression if the 5cm of the
terms is 81?

A. 12
B. 27
C. 9

ANSWERS

1. C 2. D 3. B 4. B 5. D 6. C 7. D
8. – 9. C 10. D 11. C 12. B 13. D
14. D 15. A 16. C 17. B 18. C
19. B 20. D

21. B 22. C 23. C 24. A 25. B


26. C 27. D 28. B 29. B 30. B
31. B 32. B 33. B 34. A 35. D
36. C 37. C 38. C 39. B 40. C

pdfmadeazy.com.ng
Mathematics 2019
1. Simplify (1 1 / (2 1 of32)
2 4 3. A 5.0g of salts was weighed by Tunde as 5.1g. what is
A. 3/256 B. 3/32 the percentage error?
C. 6 D. 85 A. 20 B. 2
C. 2 D. 0.2
2. If x is the addition of the prime numbers between 1 and
6, and y the H. C.F of 6,9, 15, find the product of x and 4. Find correct to one decimal place,
y 0.24633 /0.0306
A. 27 B. 30 A. 0.8 B. 1.8
C. 33 D. 90
C. 8.0 D. 8.1
5. Two sisters, Taiwo and Kehinde, own a store. The ratio 16. If g(y) = y – 3/11 + 11/ y2 – 9 what is g(y + 3)?
of Taiwo’s share to Kehind’s is 11:9. later Kehinde sells
2/3 of her share to Taiwo for #720.00. Find the value of A. y + 11 B. y + 11
the store. 11 y(y+6) 11 y(y+3)
A. #1,080.00 B. #2,400.00 C. y + 30 + 11 D. y + 3 + 11
C. #3,000.00 D. #3,600.00 11 y(y+3) 11 y(y-6)

6. A basket contains green, black and blue balls in the 17. Factorize completely (x2 + x) 2 (2x + 2)2
ratio 5:2:1. if there are 10 blue balls, find the A. (x+y)(x+2)(x-2) B. (x+ y)2(x-2)2
corresponding new ratio when 10green and 10black C. (x+1)2(x+2)2 D. (x+1)2(x+2)2(x-2)
balls are removed from the basket.
A. 1:1;1 B. 4:2:1 18. Simplify (x - y)
C. 5:1:1 D. 4:1:1 (x1/3 - y1/2)
A. x = xy + y2
2
B. x2/3 + x1/3 + y23/
th
7. A taxpayer is allowed 1/8 of his income tax free, and
pays 20% on the remainder. If he pays #490. 00 tax, C. x2/3 - x1/3 y1/3 - y2/3 D. x2 - xy + y2
what is hisincome?
A. #560.00 B. #2,450.00 19. Solve the following equation for x
C. #2,800.00 D. #3,920.00 x2 + 2x + 1 = o
r2 r1
8. Evaluate (8 1/3 x 5 2/3) / 102/3
A. 2/5 B. 5/3 A. r2 B. 1/r2
C. 25 D. 35 C. –1/r2 D. 1/r

9. If Log102 = 0.3010 and Log103 = 0.4771, evaluate, without 20. List the integral values of x which satisfy the inequality
using logarithm tableslog104.5 1 < 5 < -2x < 7
A. 0.3010 B. 0.4771 A. -1,0,1,2 B. 0,1,2,3
C. 0.6352 D. 0.9542 C. -1,0,1,2,3, D. -1,0,2,3

10. Find m such that (m ¸ 3) (1 - 3 )2 = 6 - 3 = 6 - 23 21. Given value that 3x – 5y – 3 = 0


A. 1 B. 2 2y – 6x + 5 = 0
C. 3 D. 4
the value of (x, y) is

11. The thickness of an 800-paged book is 18mm. Calculate A. (-1/8, 19/24) B. (8,24/10)
the thickness of one leaf of the book giving your answer
in metres and in standard form. C. (-8, 24/19) D. (19/24, -1/8)
A. 2.25x 10-4m B. 4.50 x 10-4m
-5
C. 2.25x 10 m D. 4.50x 10-5m 22. The solution of the quadratic equation
bx2 + qx + b = 0
12. Simplify ( x + 2) - (x - 2) A -bb 4ac B -b ppb
( x + 1) ( x +2) 2a 2a
A. 3 B. 3x + 2 C -qq2- 4bp D -qp2 - 4bp
x+1 (x+1) (x+2) 2p 2p
C. 5x+ 6 D. 2x2 + 5x + 2
(x + 1) (x + 2) (x + 1) (x + 2) 23. Simplify 1 + 1
(x2+5x+6) (x2 + 3x + 2)
13. If 1/p = (a2 + 2ab + b2) A. x+3 B. 1
(a - b) and (x+1) (x+2) (x+1) x+2) x+3)
1/q = (a + b) C. 2 D. 4
(a2 - 2ab + b2) find p/q (x+1) (x+3) (x+1) (x+3)
A. a+b B. 1
a-b a2 - b2 24. Evaluate (4a2 - 4ab2)
C. a - b D. a2 - b2 (2a2 + 5ab - 7b2)
a+b
14. If x varies inversely as the cube root of y and x = 1 when A. a -b B. 2a + 7b
y = 8 find y when x = 3 2a + b a- b
A. 1/3 B. 2/3
C. 8/27 D. 4/9 C. 2a - 7b D. 2a - 7b
a +b a-b
15. If a = -3, b = 2, c = 4, calculate (a3-b3-c1/2)
(b-1-c)
A. 37 B. –37/5
C. 37/5 D. –37
Using the graph to answer questions 25 and 26 31. S
T

y x o

Q
3 P

3xo
2
40O

1 R
In the figure above, PQ is parallel to ST and QRS = 400.
find the value of x
-4 -3 -2 -1 0 1 2 1 A. 55 B. 60
-2 C. 65 D. 75

-1 32. For which of the following exterior angles is a regular


polygon possible?
25. What is the solution of the equation i 350 ii 180 iii. 1150
x2 – x – 1 = 0? A. i and ii B. ii only
A. x= 1.6andx=-0.6 B. x= -1.6andx= 0.6 C. ii and iii D. iii only
C. x= 1.6andx=0.6 D. x= -1.6andx=-0.6
33.
26. For what values of x is the curve Q R 9cm
Y T
y = (x2 + 3) / (x + 4)
A. -3 < x< 0 B. –3 < x < 0
C. 0<x<3 D. 0<x<3

27. The solution of x2 – 2x – 1 0 are the points of intersection


of two graphs. If one of the graphs is y= 2 + x – x2, find
the second graph.
P 7cm S
A. y=1–x B. y=1+x
C. y=x–1 D. y = 3x + 3 In the figure above, PS = 7cm and RY = 9cm. If the area
of parallelogram PQRS is 56cm2, find the area of
28. If the sum of the 8th and 9th terms of an arithmetic trapezium PQTS.
progression is 72 and the 4thterm is –6, find the common A. 56cm2 B. 112cm2
2 2
difference. C. 120cm D. 176
A. 4 B. 8
C. 62/3 D. 91/3 34. A quadrilateral of a circle of radius 6cm is cut away
from each corner of a rectangle 25cm long and 18cm
29. If 7 and 189 are the first and fourth terms of a geometric wide. Find the perimeter of the remaining figure
progression respectively find the sum of the first three A. 38cm B. (38+12p)cm
terms of the progression. C. (86-12p)cm D. (86-6p)cm
A. 182 B. 91
C. 63 D. 28
35.
30.
Q
P

O
In the figure above STQ = SRP, PT = TQ = 6cm and QS
120 = 5cm. Find SR.
T A. 47/5 B. 5
100O R
C. 37/5 D. 22/5
S
36. Four interior angles o f a pentagon are 900 – x0, 900 + x0,
In the figure above, PQRS is a circle. If chords QR and 100 – 2x0, 1100 + 2x0. find the fifth interior angle.
RS are equal, calculate the value of x A. 1100 B. 1200
A. 800 B. 600 C. 130 0
D. 1400
0
C. 45 D. 400
37. 45.
O
50

60 cm

30 cm

In the figure above, PS = RS = QS and QSR = 500. find


QPR.
A. 250 B. 400
0
C. 50 D. 650 In the figure above, a solid consists of a hemisphere
surmounted by a right circular cone with radius 3.0cm
38. R and height 6.0cm. find the volume of the solid.
Z
A. 18pcm3 B. 36pcm3
3
C. 54pcm D. 108pcm3
55O
46. PQR is a triangle in which PQ= 10ccm and QPR = 600. S
45 O
is a point equidistant from P and Q. also S is a point
Y
X
equidistant from PQ and PR. If U is the foot of the
perpendicular from S on PR, find the length SU in cm to
one decimalplace.
A. 2.7 B. 2.9
P C. 3.1 D. 3.3
Q
47. In a class of 150 students, the sector in a pie chart
In the figure above, XR and YQ are tangents to the representing the students offering Physics has angle
circle YZXP if ZXR = 450 and YZX = 550 find ZYQ. 120. How many students are offering Physics?
A. 1350 B. 1250 A. 18 B. 15
0
C. 100 D. 900 C. 10 D. 5

39. From a point 143 metres away from a tree, a man 48. If x and y represents the mean and the median respectively
discovers that the angle of elevation of the tree is 300. of the following set of numbers; 11,
If the man measures this angle of elevation from a point 12,13,14,15,16,17,18,19,21,. Find x/y correct to one
2meters above the ground how high is the tree? decimal place.
A. 12m B. 14m A. 1.6 B. 1.2
C. 143m D. 16m C. 1.1 D. 1.0

40. Alero starts a 3km walk from P on a bearing 0230. she 49.
then walks 4km on a bearing 1130 to Q what is the bearing
of Q fromP?
A. 26052’ B. 5208’
C. 0
76 8’ D. 90 0 In the distribution above, the mode and the median
respectively are
41. If cot q = x/y, find cosec q A. 1.3 B. 1.2
C. 3.3 D. 0.2
A. 1/y(x2+y) B. (x/y)
50. If two dice are thrown together, what is the probability
C. 1/y(x2+y) D. y/x of obtaining at least a score of 10?
A. 1/6 B. 1/12
42. In triangle PQR, PQ = 1cm, QR = 2cm and PQR = 1200. C. 5/6 D. 11/12
Find the longest side of thetriangle
A. 3 B. 3 7/7
C. 3 7 D. 7

44. If a metal pipe 10cm long has an external diameter of


12cm and a thickness of 1cm, find the volume of the
metal used in making the pipe.
A. 120pcm3 B. 110pcm3
3
C. 60pcm D. 50pcm3
Mathematics 2020
1. Which of the following is in descending order? 13. Make R the subject of the formula
A. 9/10,4/5,3/4,17/10 B. 4/5,9/10,3/4,17/20 S = R  )
C. 6/10,17/20,4/5,3/4 D. 4/5,9/10,17/10,3/4 (3RT)
A. R = T B. T
2. Evaluate 2,700, 000 x 0.03 ¸18,000 (TS2 - 1) 2(TS2 - 1)
A. 4.5 x 100 B. 4.5 x 101
C. 4.5 x 102 D. 4.5 x 103 C R= T D. T
(TS2 + 1) 2(TS2 + 1)
3. The prime factors of 2,520 are
A. 2,9,5, B. 2,9,7, 14. Find the value of the expression
C. 2,3,5,7, D. 2,3,7,9, 32 - x364 x2 81 when x = -3/4
81 x 16
1
4. If 12 = X find x where e = 12 A. 10 / B. 101/
e 7
3
3 /8
2
– 1/
A. 20 B. 15 C. D. 13 2
C. 14 D. 12
15. The cost of dinner for a group of students is partly
5. Simplify 364r -6)1/2 cconstant and partly varies directly as the number of
students. If the cost is #74.00 when the number of
A. r B. 2r students is 20, and #96.00 when the number is 30, find
C. 1/2r D. 2/r the cost when there are 15 students.
A. #68.50 B. #63.00
6. What is the difference between 0.007685 correct to three C. #60.00 D. #52.00
significant figures and 0.007685 correct to four places
of decimal? 16. If f(x) = 2x2 + 5x + 3, find f(x + 1)
A. 10-5 B. 7 x 10-4 A. 2x2 – x B. 2x2 – x + 10
C. 8 x 10 -5
D. 10 -6 C. 4x2 + 3x + 2 D. 4x2 + 3x + 12

7. If a : b = 5: 8, x : y = 25 : 16, evaluate a/x : b/y 17. Solve the positive number x suchthat
A. 125:128 B. 3:5 2(x3 – x2 – 2x) = 1
C. 3:4 D. 2:5 A. 4 B. 3
C. 2 D. 1
8. Oke deposited #800.00 in the bank aat the rat of 121/2%
simple interest. After some time the total amount was 18. Simplify (32x - 4x2)
one and half times the principal. For how many years (2x + 18)
was the money left in the bank A. 2(x- 9) B. 2(9 + x )
A. 2 B. 4 C. 81 – x2 D. –2(x - 9)
C. 51/2 D. 8
19. Factorize completely y3 – 4xy + xy3 – 4y
9. If the surface area of a sphere is increased by 44%. A. (x + xy)(y+ 2)(y - 2)
Find the percentage increase in its diameter. B. (y + xy)(y + 2)(y - 2)
A. 44 B. 30 C. y(1 + x)(y + 2)(y - 2)
C. 22 D. 20 D. y(1 - x)(y + 2)(y - 2)

10. Simplify 4 - 1 20. If one of x3 – 8-1 is x – 2–1 , the other factors is


(2-3) A. x2 + 2-1 x– 4-1 B. x2 - 2-1 x – 4-1
A. 23 B. ............ 2., 3 C. 2 -1
x + 2 x+ 4 -1
D. x2 + 2-1 x –4-1
C. –2 + 3 D. 2, -3
21. Factorize 4a2 + 12ab – c2+ 9b2
11. Find p in terms of q if Log p + 3log q = 3 A. 4a(a – 3b) + (3b - c)2
3 3
A. (3)3 B. (q)1/3 B. (2a + 3b – c )(2a + 3b + c)
(q) (3) C. (2a – 3b -c)(2a –3b + c)
C. (q)3 D. (3)1/3 D. 4a(a – 3b) + (3b +c)2
(3) (q)
22. What are K and L respectively if ½ (3y – 4x)2 = (8x2 +
12. What are the values of y which satisfy the equation kxy + Ly2)
9y – 4 ( 3y) + 3 = 0 A. -12, 9/2 B. –6, 9
A. -1 and 0 B. –1 and 1 C. 6, 9 D. 12, 9/2
C. 1 and 3 D. 0 and 1
23. Solve the pair of equation for x and y respectively A. 1,10 B. 2,10
2x-1 – 3y-1 = 4 C. 3,13 D. 4,16
4x-1 + y-1 = 1
M
A. -1,2 B. 1,2 31.
C. 2,1 D. 2,-1
N

24. What value of Q will make the expression 4x2 + 5x + Q a


complete square?
A. 25/16 B. 25/64 Q
C. 5/8 D. 5/4 R

25. Find the range of values of r which satisfies the following MN is a tagent to the given circle at M, MR and MQ are
inequality, where a, b and c are positive. r/a+r/b+r/c >1 two chords. If QMN is 600 and MNQ is 400, find RMQ
A. r> abc B. r>abc A. 1200 B. 110
0
bc + ac + ab C. 60 D. 200

C. r > 1/a + 1/b+ 1/c D. r>1/abc 32. P

26. Express 1 - 1
(x + 1) (x - 2)
A. -3 B. 3 H K
(x +1)(2-x) (x+1)2-X)
Q R
C. -1 D. 1
(x+1)(x-2) (x+1)(x-2) In the diagram above, HK is prallel to QR, PH = 4cm and
HQ = 3cm. What is the ratio of KR;PR?
27. Simplify x - (x+ 1 ) 1/2 A. 7:3 B. 3:7
C. 3:4 D. 4:3
(x + 1) (x + 1) 1/2

A. 1 B. - 1 33. A regular polygon of (2k + 1) sides has 1400 as the size


x+1 x+ 1 of each interior angel. Find K.
A. 4 B. 41/2
C. 1 D. 1 C. 8 D. 81/2
x x +1 S
34. T
28.
y
b
a g

m O
ck h 24
P R
d f l x
Q
-1 1 2 3 i 4 5 6
If PST is a straight line and PQ = QS = SR in the above
j
e diagram, find y
A. 240 B. 480
0
C. 72 D. 840
On the curve above, the points at which the gradient of
S
the curve is equal to zeroare 35.
A. c.d.f.i.l B. b.e.g.j.m
C. a.b.c.d.f.i.j.l. D. c.d.f.h.i.l
P
29. The sum of the first two terms of a geometric progression
is x and the sum of the last two terms is y. if there are n R 60O
terms in all, then the common ratio is
A. x/y B. y/x Q
C. (x/y)1/2 D. (y/x)1/2 In the abovediagram PQ is parallel to RS and QS bisects
PQR. If PQR is 600, find x
30. If –8, m,n, 19 in arithmetic progression, find (m, n) A. 300 B. 400
0
C. 60 D. 1200

36. PQRS is a rhombus. If PR2 + QS2 = kPQ2. Determine k.


A. 1 B. 2
C. 3 D. 4
37. In DXYZ, Y = Z = 300 and XZ = 3cm find YZ 45. O
A. 3/2cm B. 33/2cm
C. 33cm D. 23cm

38. In DPQR, the bisector of QPR meets QR at S. the line PQ


is produced to V and the bisector of VQS meets PS
produced at T. if QPR = 460 and QST = 750, calculate X
QTS W
A. 410 B. 520 T
C. 64 0
D. 820 Y
Z
Y OXYZW is a pyramid with a square base such that OX
39.
= OY = OZ = OW = 5cm and XY = XW = YZ = WZ =
S 6cm. Find the height OT.
A. 25 B. 3
C. 4 D. 7

46. In preparing rice cutlets, a cook used 75g of rice, 40g of


3yO margarine, 105g of meat and 20g of bread crumbs. Find
yO
56
O

P R the angle of the sector which represents meat in a pie


Q chart.
A. If PQR is a straight line with OS = = QR, A. 300 B. 600
calculate TPQ, if QT//SR and TQS = 3y0. C. 112.50 D. 157.50
A. 620 B. 560
C. 202/ 0 D. 182/ 0 47. In a class of 30 students, the marks scored in an
3 3
examination are displayed in the following histogram.
40. R

No . of students
10
X Z
8

6
S T
4
If x : y = 5:12 and z = 52cm, find the perimeter of the
triangle.
2
A. 68cm B. 84cm
C. 100cm D. 120cm 0
20 40 60 80 100
41. The pilot of an aeroplane, flying 10km above the ground Marks scored
in the direction of a landmark, views the landmark to
have angle of depression of 350 and 550. find the distance What percentage of the students scored more than
between the two points of observation 40%
A. 10(sin 350 – sin550) A. 14% B. 40%
B. 10(cos 350 – cos 550) C. 452/ % D. 531/ %
3 3
C. 10(tan 350 –tan 550)
D. 10(cot 350 – cot550) 48. In a family of 21 people, the average age is 14years. If
the age of the grandfather is not counted, the average
42. A sin2x – 3 = 0, find x if 0 < x < 900 age drops to 12years. What is the age of the
A. 300 B. 450 grandfather?
0
C. 60 D. 900 A. 35years B. 40years
C. 42years D. 54years
43. A square tile has side 30cm. How many of these tiles
cover a rectangular floor of length 7.2cm and width 49. If n is the median and m is the mode of the following set
4.2m? of numbers,2.4,2.1,1.6,2.6,2.6,3.7,2.,1,2.6, then (n, m) is
A. 336 B. 420 A. (2.6,2.6) B. (2.5,2.6)
C. 576 D. 720 C. (2.6,2.5) D. (2.5,2.1)

44. A cylindrical metal pipe 1m long has an outer diameter 50. The numbers are chosen at random from three numbers
of 7.2cm and an inner diameter of 2.8cm. find the volume 1,3,6. find the probability that the sum of the two is not
of metal used for thecylinder. odd.
A. 440pcm3 B. 1,100pcm3 A. 2/3 B. ½
3
C. 4,400pcm D. 11,000pcm3 C. 1/3 D. 1/6
Mathematics 2021
1. Simplify (43/4 - 61/4) 12. If a = 2, b = -2 and c = -1/2,
(41/5 of 1 1/4) evaluate (ab2 – bc2) (a2c - abc)
A. 0 B. 28
A. -77/8 B. –2/7 C. –30 D. 34
C. –10/21 D. 10/21
13. Y varies inversely as x2 and X varies directly as Z2. find
2 2 2 3
2. The H.C.F. of a bx + abx and a b – b is the relationship between Y and Z, if C is a constant.
A. b B. a+b A. Z2 y = C B. Y= CZ2
C. a(a + b) D. abx (a2 – b2) C. Y = CZ 2
D. Y= C

3. Correct 241.34 (3 x 10-3)2 to 4 significant figures 14. Find the value of r in terms of p and q in the following
A. 0.0014 B. 0.001448 equation
C. 0.0022 D. 0.002172 P/2 = (r/(r+q)
A. r = q B. pq2
4. At what rate would a sum of #100.00 deposited for 5 2-p 2
2 - q2
yearsraise an interest of #7.50? C. r = p q 2 2
D. p
A. 11/ % B. 21/ % 2 - pq q(2-p)
2 2
C. 15% D. 25%
15. If f(x - 4) = x2 + 2x + 3, find f(2)
5. Three children shared a basket of mangoes in such a A. 6 B. 11
way that the first child took ¼ of the mangoes and the C. 27 D. 51
second ¾ of the remainder. What fraction of the
mangoes did the third child take? 16. Factorize 9(x + y)2 – 4(x - y)2
A. 3/16 B. 7/16 A. (x+ y) (5x+ y) B. (x+ y)2
C. 9/16 D. 13/16 C. (x+ 5y) (5x+ y) D. 5(x+y)2

6. Simplify and express in standard form 17. If a2 + b2 = 16 and 2ab = 7 find all the possible values of
(0.00275 x 0.00640/( 0.025x 0.08) (a – b )
A. 8.8 x 10-1 B. 8.8 x 102 A. 3, -3 B. 2, -2
C. 8.8 x 10-3 D. 8.8 x 103 C. 1, -1 D. 3, -1

7. Three brothers in a business deal share the profit at the 18. Divide x3 – 2x2 – 5x + 6 by (x - 1)
end of contract. The first received 1/3 of the profit and A. x2 – x –6 B. x2 – 5x + 6
the second 2/3 of the remainder. If the third received C. x2 –7x + 6 D. x2 – 5x - 6
the remaining #12.000.00, how much profit did they 19. If x + = 4, find the x2 + 1/x
share? A. 16 B. 14
A. #60,000.00 B. #54,000.00 C. 12 D. 9
C. #48,000.00 D. #42,000.00
20. What must be added to 4x2 – 4 to make it a perfect
8. Simplify 160r +  (71r + 100r
2 4 3
square?
A. -1/x2 B. 1/x2
A. 9r2 B. 12 3r C. 1 D. -1
C. 13r D. 13r
21. Find the solution of the equation
9. Simplify27 + 3/3 x – 8 x + 15 = 0
A. 43 B. 4/3 A. 3, 5 B. –3, -5
C. 33 D. 3/4 C. 9, 25 D. –9, 25

10. Simplify 3Log69 + Log612 + Log664 – Log672 22. The lengths of the sides of a right-angled triangle are
A. 5 B. 7776 xcm. (3x-1)cmand (3x + 1)cm. Find x
C. Log631 D. (7776)6 A. 5 B. 7
C. 8 D. 12
11. Simplify (1 + 1 ) -1
x-1 y-1 23. The perimeter of a rectangular lawn is 24m, if the area of
A. x/y B. xy the lawn is 35m2, how wide is the lawn?
C. y/x D. (xy)-1 A. 5m B. 7m
C. 12m D. 14m
32. The angle of a sector of a circle, radius 10.5cm, is 480.
25. Simplify x + y - x2 calculate the perimeter of the sector
(x+y) (x-y) (x2 - y2) A. 8.8cm B. 25.4cm
A. x2 B. y 2
C. 25.6cm D. 29.8cm
x2 - y 2 x2 - y2
C. x D. y 33.
x2 - y2 x2 - y2

26. Given that x2 + y2 + z2 = 194, calculate z if x = 7 and y = 3


A. 10 B. 8
C. 12.2 D. 13.4
100 O
27. Find the sum of the first twenty terms of the arithmetic
progression Log a, Log a2, Log a3
A. log a20 B. log a21
C. log a 200
D. log a210 In the figure above PS = QS and QSR = 1000, find QPR
A. 400 B. 500
0
24. A carpainter charges #40.00 per day for himself and C. 80 D. 1000
#10.00 per day for his assistant. If a fleet of a cars were
painted for #2,000.00 and the painter worked 10 days 34.
more than his assistant, how much did the assistant
receive?
A. #32.00 B. #320.00

28. Find the sum of the first 18 terms of the progression 3,


6, 12 ………..
A. 3(217 - 1) B. 3(218 ) - 1 )
18
C. 3(2 + 1) D. 3(218 - 1)

29. y

In triangle XYZ and XQP, XP= 4cm, XQ= 5cm and PQ =


QY = 3ccm. Find ZY
-1 0 2 x A. 8cm B. 6ccm
C. 4cm D. 3cm

35. Find the length of a side of a rhombus whose diagonals


are 6cm and 8cm.
A. 8cm B. 5cm
What is the equation of the quadratic function C. 4cm D. 3cm
represented by the graph above?
A. y = x2 + x - 2 B. y = x2 – x –2 36. Each of the interior angles of a regular polygon is 1400.
2
C. y = -x – x + 2 D. y = -x + x + 2 how many sides has the polygon?
30. At what value of x is the functionx2 + x + 1 minimum? A. 9 B. 8
A. -1 B. –1/2 C. 7 D. 5
C. ½ D. 1
37. S
31. Q
R

O
81 x 22
O
P T
P S Q

In the diagram above, the area of PQRS is 73.5cm2 and


its height is 10.5cm. find the length of PS if QR is one- In the figure above, PQRS is a circle. If PQT and SRT
third of PS. are straight lines, find the value of x.
A. 21cm B. 171/2 cm A. 590 B. 770
C. 14cm D. 101/2 cm C. 103 0
D. 1210
38. In a regular pentagon, PQRST, PR intersects QS at O. 44. 4 cm
calculate RQS.
A. 360 B. 720
C. 108 0
D. 1440 6 cm

39. If cos q = 12/13, find 1 + cot2 q 6 cm


A. 169/25 B. 25/169
C. 169/144 D. 144/169

40. Find the curved surface area of the frustrum in the figure.
X A. 16 10cm B. 20 10
C. 24 D.

45. The locus of a point which moves so that it is


equidistant from two intersecting straight lines is the
8 cm A. perpendicular bisector of the two lines
B. angle bisector of the two lines
C. bisector of the two lines
D. line parallel to the two lines
Y Z
46 4, 16, 30, 20, 10, 14 and 26 are represented on a piechart.
Find the sum of the angles of the sectors representing
In the figure above, YXZ = 300, XYZ = 1050 and XY = all numbers equal to or greater than 16.
8cm. CalculateYZ. A. 480 B. 840
A. 162cm B. 82cm C. 92 0
D. 2760
C. 42cm D. 22cm
47. The mean of ten positive numbers is 16. when another
41. number is added, the mean becomes 18. find the
eleventh number.
A. 3 B. 16
C. 18 D. 30

48. Below are the scores of a group of students in a test.

If the average score is 3.5, find the value of x.


A. 1 B. 2
C. 3 D. 4

In the figure above PQR is a semicircle. Calculate the 49. Twonumbers are removed at random from the numbers
area of the shaded region. 1,2,3 and 4. what is the probability that the sum of the
A. 1252/ cm2 B. 1492/ cm2 numbers removed is even?
7 7
C. 2431/7cm2 D. 2671/2 cm2 A. 2/3 B. ½
C. 1/3 D. ¼
42. A cylindrical pipe, made of metal is 3cm, thick if the
internal radius of the pipe is 10cm. Find the volume of 50. Find the probability that a number selected at random
metal used in making 3m of the pipe from 41 to 56 is a multiple of 9
A. 153cm3 B. 207cm3 A. 1/9 B. 2/15
3
C. 15,300cm D. 20,700cm3 C. 3/16 D. 7/8

43. If the height of two circular cylinders are in the ratio 2:3
and their base radii are in the ratio 9. what is the ratio of
their volume
A. 27:32 B. 27:23
C. 23:32 D. 21:27

You might also like